Vous êtes sur la page 1sur 53

TAXATION 1 | B2015

CASE DIGESTS

NATIONAL DEVELOPMENT
COMPANY vs.
COMMISSIONER OF INTERNAL
REVENUE
June 30, 1987
CRUZ, J.
Mica Maurinne M. Adao
Topic: Passive Income- Interest Income

SUMMARY: NDC entered into contracts in Tokyo with


several
Japanese
shipbuilding
companies
for
the
construction of 12 ocean-going vessels. The balance
payment was embodied in a promissory note which earns
5% per annum interest. NDC remitted to the shipbuilding
companies the interest on the balance payment without
withholding the necessary tax so BIR served a warrant of
distraint and levy to enforce collection. CTA sustained BIR
ruling. SC affirmed. Clearly, the interest remitted to the
Japanese shipbuilders on the unpaid balance of the purchase
price of the vessels acquired by NDC is interest derived from
sources within the Philippines subject to income tax under
the then Section 24(b)(1) of the NIRC.

DOCTRINE: Under the terms of the law, that the


Government's right to levy and collect income tax on
interest received by foreign corporations not engaged in
trade or business within the Philippines is not planted upon
the condition that 'the activity or labor and the sale from
which the (interest) income flowed had its situs' in the
Philippines. The law specifies: 'Interest derived from sources
within the Philippines, and interest on bonds, notes, or other
interest-bearing obligations of residents, corporate or
otherwise.' Nothing there speaks of the 'act or activity' of

non-resident corporations in the Philippines, or place where


the contract is signed.

The residence of obligor who pays the interest rather than


the physical location of the securities, bonds or notes or the
place of payment, is the determining factor of the source of
interest income. Accordingly, if the obligor is a resident of
the Philippines the interest payment paid by him can have
no other source than within the Philippines. The interest is
paid not by the bond, note or other interest-bearing
obligations, but by the obligor.

FACTS:
The National Development Company (NDC) entered into
contracts in Tokyo with several Japanese shipbuilding
companies for the construction of 12 ocean-going vessels.
The purchase price was to come from the proceeds of bonds
issued by the Central Bank. Initial payments were made in
cash and through irrevocable letters of credit.
14
promissory notes were signed for the balance by the NDC
and, as required by the shipbuilders, guaranteed by the
Republic of the Philippines. Pursuant thereto, the remaining
payments and the interests thereon were remitted in due
time by the NDC to Tokyo. The vessels were eventually
completed and delivered to the NDC in Tokyo.
The NDC remitted to the shipbuilders in Tokyo the total
amount of US$4,066,580.70 as interest (rate: 5% per
annum) on the balance of the purchase price. No tax was
withheld. The Commissioner then held the NDC liable on
such tax in the total sum of P5,115,234.74. Negotiations
followed but failed. The BIR thereupon served on the NDC a
warrant of distraint and levy to enforce collection of the
claimed amount.

TAXATION 1 | B2015
CASE DIGESTS

The NDC went to the CTA which sustained BIR ruling except
for a slight reduction of the tax deficiency in the sum of
P900.00, representing the compromise penalty. Thus, this
petition for certiorari.
ISSUE: Whether or not the interest on the balance
price is subject to withholding tax?
RULING: YES
RATIO:
The Japanese shipbuilders were liable to tax on the interest
remitted to them under Section 37 of the Tax Code, thus:
SEC. 37. Income from sources within the Philippines.
(a) Gross income from sources within the
Philippines. The following items of gross income
shall be treated as gross income from sources within
the Philippines:
(1) Interest. Interest derived from sources within
the Philippines, and interest on bonds, notes, or
other interest-bearing obligations of residents,
corporate or otherwise;
xxx xxx xxx
The petitioner argues that the Japanese shipbuilders were
not subject to tax under the above provision because all the
related activities the signing of the contract, the
construction of the vessels, the payment of the stipulated
price, and their delivery to the NDC were done in Tokyo.
The law, however, does not speak of activity but of "source,"
which in this case is the NDC. This is a domestic and
resident corporation with principal offices in Manila.

the Philippines, it is a domestic corporation, resident of the


Philippines. (Sec. 84(c), NIRC.) The interest paid by NDC,
which is admittedly a resident of the Philippines, is on the
promissory notes issued by it. Clearly, therefore, the
interest remitted to the Japanese shipbuilders in
Japan in 1960, 1961 and 1962 on the unpaid balance
of the purchase price of the vessels acquired by NDC
is interest derived from sources within the
Philippines subject to income tax under the then
Section 24(b)(1) of the NIRC.
There is no basis for saying that the interest payments were
obligations of the Republic of the Philippines and that the
promissory notes of the NDC were government securities
exempt from taxation under Section 29(b)[4] of the Tax
Code, reading as follows:
SEC. 29. Gross Income. xxxx xxx xxx xxx
(b) Exclusion from gross income. The following
items shall not be included in gross income and shall
be exempt from taxation under this Title:
xxx xxx xxx
(4) Interest on Government Securities. Interest
upon the
obligations of the Government of the Republic of the
Philippines or any political subdivision thereof, but in
the case of such obligations issued after approval of
this Code, only to the extent provided in the act
authorizing the issue thereof. (As amended by
Section 6, R.A. No. 82)

**See Doctrine

The law invoked by NDC as authorizing the issuance of


securities is R.A. No. 1407, which in fact is silent on this
matter. C.A. No. 182 as amended by C.A. No. 311 does carry
such authorization but, like R.A. No. 1407, does not exempt
from taxes the interests on such securities.

The law is clear.The residence of the obligor (NDC) which


paid the interest under consideration, petitioner herein, is
Calle Pureza, Sta. Mesa, Manila, Philippines; and as a
corporation duly organized and existing under the laws of

It is also incorrect to suggest that the Republic of the


Philippines could not collect taxes on the interest remitted
because of the undertaking signed by the Secretary of
Finance in each of the promissory notes that:

TAXATION 1 | B2015
CASE DIGESTS

Upon authority of the President of the Republic of the


Philippines, the undersigned, for value received,
hereby absolutely and unconditionally guarantee
(sic), on behalf of the Republic of the Philippines, the
due and punctual payment of both principal and
interest of the above note.
There is nothing in the above undertaking exempting the
interests from taxes. NDC has not established a clear waiver
therein of the right to tax interests. Tax exemptions cannot
be merely implied but must be categorically and
unmistakably expressed.
Any doubt concerning this
question must be resolved in favor of the taxing power.
Nowhere in the said undertaking do we find any inhibition
against the collection of the disputed taxes. In fact, such
undertaking was made
by the government in consonance with and certainly not
against Sec. 53(b), Sec 54 of the NIRC. Manifestly, the said
undertaking of the Republic of the Philippines merely
guaranteed the obligations of the NDC but without
diminution of its taxing power under existing laws.
It is not the NDC that is being taxed. The tax was due on the
interests earned by the Japanese shipbuilders. It was the
income of these companies and not the Republic of the
Philippines that was subject to the tax the NDC did not
withhold. In effect, therefore, the imposition of the
deficiency taxes on the NDC is a penalty for its failure to
withhold the same from the Japanese shipbuilders. Such
liability is imposed by Section 53(c) of the Tax Code
Philippine Guaranty Co. v. The Commissioner of Internal
Revenue:
In case of doubt, a withholding agent may always protect
himself by withholding the tax due, and promptly causing a
query to be addressed to the CIR for the determination
whether or not the income paid to an individual is not
subject to withholding. In case the CIR decides that the
income paid to an individual is not subject to withholding,

the withholding agent may thereupon remit the amount of a


tax withheld.
Strict observance of said steps is required of a withholding
agent before he could be released from liability. Generally,
the law frowns upon exemption from taxation; hence, an
exempting provision should be construed strictissimi juris.
The petitioner was remiss in the discharge of its obligation
as the withholding agent of the government and so should
be held liable for its omission.
WHEREFORE, the appealed decision is AFFIRMED

CIR V. SC JOHNSON AND


SON, INC.
1999
Gonzaga-Reyes, J:
Jayson C. Aguilar

SUMMARY: Respondent, SC Johnson and Son, Inc., entered


into a license agreement with SC Johnson and Son, USA (a
non-resident foreign corporation based in the U.S.A.)
pursuant to which the respondent was granted the right to
use the trademark, patents and technology owned by the
latter including the right to manufacture, package and
distribute the products covered by the Agreement and
secure assistance in management, marketing and
production from SC Johnson and Son, U. S. A. For the use of
the trademark or technology, respondent was obliged to
pay SC Johnson and Son USA royalties based on a
percentage of net sales. Respondent subjected the same to
25% withholding tax on royalty payments. Respondent
later claimed for refund of overpaid withholding tax on
royalties arguing that SC Johnson and Son USA is only

TAXATION 1 | B2015
CASE DIGESTS

subject to 10% withholding tax pursuant to the mostfavored nation clause of the RP-US Tax Treaty in relation to
the RP-West Germany Tax Treaty [Article 12 (2) (b)]. SC
held that the respondent is not entitled to a refund.
DOCTRINE:
The
phrase
paid
under
similar
circumstances under the most favored nation clause [Sec.
13(2)(b)(iii)] of the RP-US Tax Treaty refers to the payment
of taxes and not the payment of the subject matter of the
tax i.e. royalties.
FACTS: Respondent, SC Johnson and Son, Inc., a domestic
corporation organized and operating under the Philippine
laws, entered into a license agreement with SC Johnson and
Son, United States of America (USA), a non-resident foreign
corporation based in the U.S.A. pursuant to which the
respondent was granted the right to use the trademark,
patents and technology owned by the latter including the
right to manufacture, package and distribute the products
covered by the Agreement and secure assistance in
management, marketing and production from SC Johnson
and Son, U. S. A.
The said License Agreement was duly registered with the
Technology Transfer Board of the Bureau of Patents, Trade
Marks and Technology Transfer. For the use of the trademark
or technology, respondent was obliged to pay SC Johnson
and Son, USA royalties based on a percentage of net sales.
Respondent subjected the same to 25% withholding tax on
royalty payments which respondent paid for the period
covering July 1992 to May 1993 in the total amount
of P1,603,443.00
On October 29, 1993, respondent filed with the International
Tax Affairs Division (ITAD) of the BIR a claim for refund of
overpaid withholding tax on royalties arguing that, SC
Johnson and Son, USA is only subject to 10% withholding tax
pursuant to the most-favored nation clause of the RP-US Tax

Treaty [Article 13 Paragraph 2 (b) (iii)] in relation to the RPWest Germany Tax Treaty [Article 12 (2) (b)].
Respondent claim for the refund of P963,266.00.
The Commissioner did not act on the said claim for refund.
Thus, respondent filed a petition for review before the Court
of Tax Appeals.
CTA: In favor of respondent
CA: Affirm CTA ruling in toto.
ISSUES: Whether the phrase paid under similar
circumstances in Art. 13(2)(b)(iii) of the RP-US Tax
Treaty refers to the payment of tax or the payment of
royalties
Note: Art. 13(2)(b)(iii) of the RP-US Tax Treaty reads:
1) Royalties derived by a resident of one of the Contracting States
from sources within the other Contracting State may be taxed by
both Contracting States.
2) However, the tax imposed by that Contracting State shall not
exceed.
a) In the case of the United States, 15 percent of the gross
amount of the royalties, and
b) In the case of the Philippines, the least of:
(i) 25 percent of the gross amount of the royalties;
(ii) 15 percent of the gross amount of the royalties, where the
royalties are paid by a corporation registered with the
Philippine Board of Investments and engaged in preferred
areas of activities; and
(iii) the lowest rate of Philippine tax that may be imposed on
royalties
of the same kind
paid
under
similar
circumstances to a resident of a third State.

Argument of CIR: Under the RP-West Germany Tax Treaty,


the Philippine tax paid on income from sources within the
Philippines is allowed as a credit against German income
and corporation tax on the same income. In the case of
royalties for which the tax is reduced to 10 or 15 percent
according to paragraph 2 of Article 12 of the RP-West

TAXATION 1 | B2015
CASE DIGESTS

Germany Tax Treaty, the credit shall be 20% of the gross


amount of such royalty. To illustrate, the royalty income of a
German resident from sources within the Philippines arising
from the use of, or the right to use, any patent, trade mark,
design or model, plan, secret formula or process, is taxed at
10% of the gross amount of said royalty under certain
conditions. The rate of 10% is imposed if credit against the
German income and corporation tax on said royalty is
allowed in favor of the German resident. That means the
rate of 10% is granted to the German taxpayer if he is
similarly granted a credit against the income and
corporation tax of West Germany. The clear intent of the
matching credit is to soften the impact of double taxation
by different jurisdictions. The RP-US Tax Treaty contains no
similar matching credit as that provided under the RP-West
Germany Tax Treaty. Hence, the tax on royalties under the
RP-US Tax Treaty is not paid under similar circumstances as
those
obtaining
in
the
RP-West
Germany
Tax
Treaty. Therefore, the most favored nation clause in the
RP-West Germany Tax Treaty cannot be availed of in
interpreting the provisions of the RP-US Tax Treaty.
Argument of Respondent SC Johnson: The phrase paid
under similar circumstances does not refer to payment of
the tax but to the subject matter of the tax, that is,
royalties, because the most favored nation clause is
intended to allow the taxpayer in one state to avail of more
liberal provisions contained in another tax treaty wherein
the country of residence of such taxpayer is also a party
thereto, subject to the basic condition that the subject
matter of taxation in that other tax treaty is the same as
that in the original tax treaty under which the taxpayer is
liable; thus, the RP-US Tax Treaty speaks of royalties of the
same kind paid under similar circumstances.
RULING: The phrase paid under similar circumstances
under Sec. 13(2)(b)(iii) of the RP-US Tax Treaty refers to the
payment of taxes and not the payment of the subject matter
of the tax i.e. royalties.

The SC was unable to sustain the position of the CTA and


the CA that the phrase paid under similar circumstances in
Art. 13 (2) (b) (iii) of the RP-US Tax Treaty should be
interpreted to refer to payment of royalty, and not to the
payment of the tax, for the reason that the phrase paid
under similar circumstances is followed by the phrase to a
resident of a third state. The CTA held that Words are to
be understood in the context in which they are used, and
since what is paid to a resident of a third state is not a tax
but a royalty logic instructs that the treaty provision in
question should refer to royalties of the same kind paid
under similar circumstances.
The construction of the CTA and CA is based principally on
syntax or sentence structure but fails to take into account
the purpose animating the treaty provisions in point. To
begin with, there is no law or rule pertinent to the payment
of royalties which provides for the payment of royalties
under dissimilar circumstances. The tax rates on royalties
and the circumstances of payment thereof are the same for
all the recipients of such royalties and there is no disparity
based on nationality in the circumstances of such
payment. On the other hand, a cursory reading of the
various tax treaties will show that there is no similarity in
the provisions on relief from or avoidance of double
taxation as this is a matter of negotiation between the
contracting parties. This dissimilarity is true particularly in
the treaties between the Philippines and the United States
and between the Philippines and West Germany.
The RP-US Tax Treaty is just one of a number of bilateral
treaties which the Philippines has entered into for the
avoidance of double taxation. The purpose of these
international agreements is to reconcile the national fiscal
legislations of the contracting parties in order to help the
taxpayer avoid simultaneous taxation in two different
jurisdictions. More precisely, the tax conventions are drafted
with a view towards the elimination of international juridical
double taxation, which is defined as the imposition of
comparable taxes in two or more states on the same

TAXATION 1 | B2015
CASE DIGESTS

taxpayer in respect of the same subject matter and for


identical periods. The apparent rationale for doing away with
double taxation is to encourage the free flow of goods and
services and the movement of capital, technology and
persons between countries, conditions deemed vital in
creating robust and dynamic economies.
In negotiating tax treaties, the underlying rationale for
reducing the tax rate is that the Philippines will give up a
part of the tax in the expectation that the tax given up for
this particular investment is not taxed by the other
country. Thus the petitioner correctly opined that the phrase
royalties paid under similar circumstances in the most
favored nation clause of the US-RP Tax Treaty necessarily
contemplated circumstances that are tax-related.
Given the purpose underlying tax treaties and the rationale
for the most favored nation clause, the concessional tax rate
of 10 percent provided for in the RP-Germany Tax Treaty
should apply only if the taxes imposed upon royalties in the
RP-US Tax Treaty and in the RP-Germany Tax Treaty are paid
under similar circumstances. This would mean that
respondent must prove that the RP-US Tax Treaty grants
similar tax reliefs to residents of the United States in respect
of the taxes imposable upon royalties earned from sources
within the Philippines as those allowed to their German
counterparts under the RP-Germany Tax Treaty. If the state
of residence does not grant some form of tax relief to the
investor, no benefit would redound to the Philippines, i.e.,
increased investment resulting from a favorable tax regime,
should it impose a lower tax rate on the royalty earnings of
the investor, and it would be better to impose the regular
rate rather than lose much-needed revenues to another
country.
The RP-US and the RP-West Germany Tax Treaties do not
contain similar provisions on tax crediting. Article 24 of the
RP-Germany Tax Treaty expressly allows crediting against
German income and corporation tax of 20% of the gross
amount of royalties paid under the law of the

Philippines. On the other hand, Article 23 of the RP-US Tax


Treaty, which is the counterpart provision with respect to
relief for double taxation, does not provide for similar
crediting of 20% of the gross amount of royalties paid. Thus,
the concessional tax rate of 10 percent provided for in the
RP-Germany Tax Treaty cannot be applied to the most
favored nation clause under the RP-US Tax Treaty.
DISPOSITIVE: CA reversed.
respondent disallowed.

Petition

for

refund

by

CIR v. CA
Nov. 20, 1944
Mahoney, J.
Dave Anastacio

SUMMARY: Don Andres Soriano formed ANSCOR, of which


he owned shares of stock. Don Andres died, and ANSCOR
redeemed shares of stock from the estate of Don Andres.
The CIR held ANSCOR liable for taxes. The CTA reversed,
and CA affirmed the reversal. The SC held that they were
liable.
DOCTRINE: The general rule states that: A stock
dividend representing the transfer of surplus to
capital account shall not be subject to tax. Stock
dividends, strictly speaking, represent capital and do not
constitute income to its recipient. So that the mere
issuance thereof is not yet subject to income tax as they
are nothing but an "enrichment through increase in value
of capital investment." The exception provides that the
redemption or cancellation of stock dividends,
depending on the "time" and "manner" it was made,
is essentially equivalent to a distribution of taxable
dividends," making the proceeds thereof "taxable
income" "to the extent it represents profits". The
exception was designed to prevent the issuance and
cancellation or redemption of stock dividends, which is

TAXATION 1 | B2015
CASE DIGESTS

fundamentally not taxable, from being made use of as a


device for the actual distribution of cash dividends, which
is taxable. For the exempting clause of Section, 83(b) to
apply, it is indispensable that: (a) there is redemption or
cancellation; (b) the transaction involves stock dividends
and (c) the "time and manner" of the transaction makes it
"essentially equivalent to a distribution of taxable
dividends."
FACTS: (I dont think the numbers/other facts are important
at all. Focus on the redemption) Don Andres Soriano, a
citizen and resident of the United States, formed the
corporation A. Soriano Y Cia, predecessor of ANSCOR with
a P1,000,000.00 capitalization divided into 10,000 common
shares at a par value of P100/share. ANSCOR is wholly
owned and controlled by the family of Don Andres, who are
all non-resident aliens. In 1937, Don Andres subscribed to
4,963 shares of the 5,000 shares originally issued.
ANSCORs authorized capital stock was increased to
P2,500,000.00 divided into 25,000 common shares with the
same par value. Of the additional 15,000 shares, only
10,000 was issued which were all subscribed by Don Andres,
after the other stockholders waived in favor of the former
their pre-emptive rights to subscribe to the new issues. This
increased his subscription to 14,963 common shares. A
month later, Don Andres transferred 1,250 shares each to
his two sons, Jose and Andres Jr., as their initial investments
in ANSCOR.
By 1947, ANSCOR declared stock dividends. Other
stock dividend declarations were made between 1949 and
December 20, 1963. On December 30, 1964 Don Andres
died. As of that date, the records revealed that he had a
total shareholdings of 185,154 shares. 50,495 of which are
original issues and the balance of 134,659 shares as stock
dividend declarations. Correspondingly, one-half of that
shareholdings or 92,577 shares were transferred to his wife,
Doa Carmen Soriano, as her conjugal share. The offer half

formed part of his estate.


A day after Don Andres died, ANSCOR increased its
capital stock to P20M and in 1966 further increased it to
P30M. In the same year (December 1966), stock dividends
worth 46,290 and 46,287 shares were respectively received
by the Don Andres estate and Doa Carmen from ANSCOR.
Hence, increasing their accumulated shareholdings to
138,867 and 138,864 common shares each.
On December 28, 1967, Doa Carmen requested a
ruling from the United States Internal Revenue Service (IRS),
inquiring if an exchange of common with preferred shares
may be considered as a tax avoidance scheme. By January
2, 1968, ANSCOR reclassified its existing 300,000 common
shares into 150,000 common and 150,000 preferred shares.
In a letter-reply dated February 1968, the IRS opined
that the exchange is only a recapitalization scheme and not
tax avoidance. Consequently, on March 31, 1968 Doa
Carmen exchanged her whole 138,864 common shares for
138,860 of the preferred shares. The estate of Don Andres in
turn exchanged 11,140 of its common shares for the
remaining 11,140 preferred shares.
On June 30, 1968, pursuant to a Board
Resolution, ANSCOR redeemed 28,000 common
shares from the Don Andres' estate. By November
1968, the Board further increased ANSCOR's capital stock to
P75M divided into 150,000 preferred shares and 600,000
common shares. About a year later, ANSCOR again
redeemed 80,000 common shares from the Don
Andres' estate, further reducing the latter's common
shareholdings to 19,727. As stated in the Board
Resolutions, ANSCOR's business purpose for both
redemptions of stocks is to partially retire said stocks
as treasury shares in order to reduce the company's
foreign exchange remittances in case cash dividends
are declared.

TAXATION 1 | B2015
CASE DIGESTS

In 1973, after examining ANSCORs books of account


and record Revenue examiners issued a report proposing
that ANSCOR be assessed for deficiency withholding tax-atsource, for the year 1968 and the 2nd quarter of 1969 based
on the transaction of exchange and redemption of stocks.
BIR made the corresponding assessments. ANSCORs
subsequent protest on the assessments was denied in 1983
by petitioner. ANSCOR filed a petition for review with the
CTA, the Tax Court reversed the ruling. CA affirmed the
ruling of the CTA. Hence this position.
ISSUES: WON ANSCOR's redemption of stocks from its
stockholder can be considered as "essentially
equivalent to the distribution of taxable dividend"
making the proceeds taxable- YES
RULING: It is taxable
RATIO: The bone of contention is the interpretation and
application of Section 83(b) of the 1939 Revenue Act 38
which provides:
Sec. 83. Distribution of dividends or assets by
corporations. (b) Stock dividends A stock dividend
representing the transfer of surplus to capital account shall
not be subject to tax. However, if a corporation cancels or
redeems stock issued as a dividend at such time and in such
manner as to make the distribution and cancellation or
redemption, in whole or in part, essentially equivalent to the
distribution of a taxable dividend, the amount so distributed
in redemption or cancellation of the stock shall be
considered as taxable income to the extent it represents a
distribution of earnings or profits accumulated after March
first, nineteen hundred and thirteen.
Sec. 83(b) of the 1939 NIRC was taken from the
Section 115(g)(1) of the U.S. Revenue Code of 1928. It laid
down the general rule known as the proportionate test
wherein stock dividends once issued form part of the capital
and, thus, subject to income tax. Specifically, the general
rule states that: A stock dividend representing the

transfer of surplus to capital account shall not be


subject to tax. Stock dividends, strictly speaking,
represent capital and do not constitute income to its
recipient. So that the mere issuance thereof is not yet
subject to income tax as they are nothing but an
"enrichment through increase in value of capital
investment."
The exception provides that the redemption or
cancellation of stock dividends, depending on the "time"
and "manner" it was made, is essentially equivalent to a
distribution of taxable dividends," making the proceeds
thereof "taxable income" "to the extent it represents
profits". The exception was designed to prevent the
issuance and cancellation or redemption of stock dividends,
which is fundamentally not taxable, from being made use of
as a device for the actual distribution of cash dividends,
which is taxable.
Simply put, depending on the circumstances, the
proceeds of redemption of stock dividends are essentially
distribution of cash dividends, which when paid becomes the
absolute property of the stockholder. Thereafter, the latter
becomes the exclusive owner thereof and can exercise the
freedom of choice. Having realized gain from that
redemption, the income earner cannot escape income tax.
For the exempting clause of Section, 83(b) to apply, it is
indispensable that: (a) there is redemption or cancellation;
(b) the transaction involves stock dividends and (c) the
"time and manner" of the transaction makes it "essentially
equivalent to a distribution of taxable dividends."
Redemption is repurchase, a reacquisition of stock by
a corporation which issued the stock 89 in exchange for
property, whether or not the acquired stock is cancelled,
retired or held in the treasury. Essentially, the corporation
gets back some of its stock, distributes cash or property to
the shareholder in payment for the stock, and continues in
business as before. In the case, ANSCOR redeemed shares
twice. But where did the shares redeemed come from? If its

TAXATION 1 | B2015
CASE DIGESTS

source is the original capital subscriptions upon


establishment of the corporation or from initial capital
investment in an existing enterprise, its redemption to the
concurrent value of acquisition may not invite the
application of Sec. 83(b) under the 1939 Tax Code, as it is
not income but a mere return of capital. On the contrary, if
the redeemed shares are from stock dividend declarations
other than as initial capital investment, the proceeds of the
redemption is additional wealth, for it is not merely a return
of capital but a gain thereon.

with the wisdom of these purposes but on their relevance to


the whole transaction which can be inferred from the
outcome thereof. It is the "net effect rather than the motives
and plans of the taxpayer or his corporation". The test of
taxability under the exempting clause, when it provides
"such time and manner" as would make the redemption
"essentially equivalent to the distribution of a taxable
dividend", is whether the redemption resulted into a flow of
wealth. If no wealth is realized from the redemption, there
may not be a dividend equivalence treatment.

It is not the stock dividends but the proceeds of its


redemption that may be deemed as taxable dividends. At
the time of the last redemption, the original common shares
owned by the estate were only 25,247.5 91 This means that
from the total of 108,000 shares redeemed from the estate,
the balance of 82,752.5 (108,000 less 25,247.5) must have
come from stock dividends. In the absence of evidence to
the contrary, the Tax Code presumes that every distribution
of corporate property, in whole or in part, is made out of
corporate profits such as stock dividends. The capital cannot
be distributed in the form of redemption of stock dividends
without violating the trust fund doctrine.

The test of taxability under the exempting clause of


Section 83(b) is, whether income was realized through the
redemption of stock dividends. The redemption converts
into money the stock dividends which become a realized
profit or gain and consequently, the stockholder's separate
property. Profits derived from the capital invested cannot
escape income tax. As realized income, the proceeds of the
redeemed stock dividends can be reached by income
taxation regardless of the existence of any business purpose
for the redemption. Otherwise, to rule that the said proceeds
are exempt from income tax when the redemption is
supported by legitimate business reasons would defeat the
very purpose of imposing tax on income.

With respect to the third requisite, ANSCOR


redeemed stock dividends issued just 2 to 3 years earlier.
The time alone that lapsed from the issuance to the
redemption is not a sufficient indicator to determine
taxability. It is a must to consider the factual circumstances
as to the manner of both the issuance and the redemption.
The issuance of stock dividends and its subsequent
redemption must be separate, distinct, and not related, for
the redemption to be considered a legitimate tax scheme.
Redemption cannot be used as a cloak to distribute
corporate earnings.
ANSCOR invoked two reasons to justify the
redemptions (1) the alleged "filipinization" program and
(2) the reduction of foreign exchange remittances in case
cash dividends are declared. The Court is not concerned

The issuance and the redemption of stocks are two


different transactions. Although the existence of legitimate
corporate purposes may justify a corporation's acquisition of
its own shares under Section 41 of the Corporation Code,
such purposes cannot excuse the stockholder from the
effects of taxation arising from the redemption.
Even if the said purposes support the redemption
and justify the issuance of stock dividends, the same has no
bearing whatsoever on the imposition of the tax herein
assessed because the proceeds of the redemption are
deemed taxable dividends since it was shown that income
was generated therefrom.
The proceeds thereof are essentially considered

TAXATION 1 | B2015
CASE DIGESTS

equivalent to a distribution of taxable dividends. As "taxable


dividend" under Section 83(b), it is part of the "entire
income" subject to tax under Section 22 in relation to
Section 21 120 of the 1939 Code. Moreover, under Section
29(a) of said Code, dividends are included in "gross income".
As income, it is subject to income tax which is required to be
withheld at source.

unequivocably to hold that the proceeds of life insurance


policies received by corporations constitute income, which
is taxable. Life insurance is like that of fire and marine
insurance a contract of indemnity. Proceeds of life
insurance, payable upon the death of the insured, are
considered as indemnity rather than income to the hairs or
beneficiaries who could be corporations or individuals.

DISPOSITIVE: The Court concludes that the $350,000 of


the $410,000 attributable to the suit is thus taxable income.

FACTS:
El Oriente in order to protect itself against the loss
that it might suffer by reason of the death of its
manager, A. Velhagen, who had had more than
thirty-five (35) years of experience in the
manufacture of cigars in the Philippines, procured
from the Manufacturers Life Insurance Co., of
Toronto, Canada, thru its local agent E. E. Elser, an
insurance policy on the life of the said A. Velhagen
for the sum of $50,000, United States currency
designating itself as the beneficiary.
El Oriente paid for the premiums due thereon and
charged as expenses of its business all the said
premiums and deducted the same from its gross
incomes as reported in its annual income tax returns,
which deductions were allowed upon a showing that
such premiums were legitimate expenses of its
business.
Upon the death of A. Velhagen in 1929, the El Oriente
received all the proceeds of the said life insurance
policy, together with the interests and the dividends
accruing thereon, aggregating P104,957.88
CIR assessed El Oriente for deficiency taxes because
El Oriente did not include as income the proceeds
received from the insurance.

El Oriente v. Posadas
September 21, 1931
Malcolm
alycat

SUMMARY: El Oriente procured from the Manufacturers


Life Insurance, an insurance policy on the life of its
manager, A. Velhagen, designating itself as the beneficiary.
El Oriente paid for the premiums due thereon and charged
as expenses of its business all the said premiums and
deducted the same from its gross incomes as reported in
its annual income tax returns, which deductions were
allowed upon a showing that such premiums were
legitimate expenses of its business. Upon the death of A.
Velhagen, El Oriente received all the proceeds of the said
life insurance policy, together with the interests and the
dividends accruing thereon. CIR assessed El Oriente for
deficiency taxes because El Oriente did not include as
income the proceeds received from the insurance. CFI held
such as taxable. SC reversed.
DOCTRINE: It is certain that the proceeds of life insurance
policies paid to individual beneficiaries upon the death of
the insured are exempt. It is not so certain that the
proceeds of life insurance policies paid to corporate
beneficiaries upon the death of the insured are likewise
exempt. But at least, it may be said that the law is
indefinite in phraseology and does not permit the court

ISSUE:
Whether or not the proceeds of insurance taken by a
corporation on the life of an important official to indemnify it
against loss in case of his death, are taxable as income
under the Philippine Income Tax Law

10

TAXATION 1 | B2015
CASE DIGESTS

HELD: NOT TAXABLE.


RATIO:
In Chapter I of the Tax Code, is to be found section 4 which
provides that, "The following incomes shall be exempt from
the provisions of this law: (a) The proceeds of life insurance
policies paid to beneficiaries upon the death of the insured .
. ." Section 10, as amended, in Chapter II On Corporations,
provides that, "There shall be levied, assessed, collected,
and paid annually upon the total net income received in the
preceding calendar year from all sources by every
corporation . . .a tax of three per centum upon such
income . . ." Section 11 in the same chapter, provides the
exemptions under the law, but neither here nor in any other
section is reference made to the provisions of section 4 in
Chapter I.
Under the view we take of the case, it is sufficient for our
purposes to direct attention to the anomalous and vague
condition of the law. It is certain that the proceeds of life
insurance policies paid to individual beneficiaries upon the
death of the insured are exempt. It is not so certain that the
proceeds of life insurance policies paid to corporate
beneficiaries upon the death of the insured are likewise
exempt. But at least, it may be said that the law is indefinite
in phraseology and does not permit us unequivocally to hold
that the proceeds of life insurance policies received by
corporations constitute income which is taxable
It will be recalled that El Oriente, took out the insurance on
the life of its manager, who had had more than thirty-five
years' experience in the manufacture of cigars in the
Philippines, to protect itself against the loss it might suffer
by reason of the death of its manager. We do not believe
that this fact signifies that when the plaintiff received
P104,957.88 from the insurance on the life of its manager, it
thereby realized a net profit in this amount. It is true that
the Income Tax Law, in exempting individual beneficiaries,
speaks of the proceeds of life insurance policies as income,

but this is a very slight indication of legislative intention. In


reality, what the plaintiff received was in the nature of an
indemnity for the loss which it actually suffered because of
the death of its manager.
DISPOSITIVE: The foregoing pronouncement will result in
the judgment being REVERSED and in another judgment
being rendered in favor of the plaintiff and against the
defendant for the sum of P3,148.74. So ordered, without
costs in either instance.

CIR v. COA
January 29,1993
Narvasa, C.J.
Sai (sorry the provisions are long)

SUMMARY: Savellano furnished BIR with information


denouncing NCA and PNCC for non-payment of tax totaling
P234M. BIR demanded from NCA (P15M) and PNOC (P93M).
After recommendation to Minister of FIncance, he was
given informers reward equivalent to 15% (P2M for NCA)
and (14M for PNOC). COA rendered a decision disallowing
audit in payment on ground that under section 281 of NIRC
such is conditioned upon actual recovery or collection of
revenues.
DOCTRINE: COA's disallowance of the informer's reward
on the ground that there was actually no revenue realized
or recovered as 2 government agencies were involved
overlooks the fact that NCA and PNOC, possess legal
personalities separate and distinct from the Philippine
government. Although both are GOCCs, they perform
proprietary functions. Their revenues do not automatically
devolve to the general coffers of the government. Unless
transferred to the Philippine government through the
vehicle of taxation, no part of their revenues is available
for appropriation by the Legislature for expenditure in
government projects; such revenues remain said agencies'
in their entirety, to be applied to and expended for their

11

TAXATION 1 | B2015
CASE DIGESTS

own exclusive purpose. When revenues are subjected to


tax, the portion thereof corresponding to such tax
becomes, in its own, revenue or the government accruing
to the General Fund.
FACTS: Tirso B. Savellano furnished the BIR with a
confidential affidavit of information denouncing the National
Coal Authority (NCA) and the Philippine National Oil
Company (PNOC) for non- payment of taxes totalling P234
Million on interest earnings of their respective money
placements with PNB since October 15, 1984. Investigation
by the BIR confirmed the reported tax liabilities, and upon
demands thereafter made, NCA and PNOC paid to the BIR
the P15,986,165 and P93,955,479.12. BIR Commissioner
Bienvenido Tan, Jr. recommended to the Minister of Finance
payment to Savellano of an informer's reward equivalent to
15% of the amount of P15,986,165.00 paid by NCA, or
P2,397,924.75. Savellano was paid the said amount.
Savellano was paid his informer's reward in the PNOC case
in the total amount of P14,093,321.89 in 4 installments.
COA rendered COA Decision No. 740 disallowing in audit the
payment of informer's reward to Savellano in the NCA case
on the ground that payment of an informer's reward under
Section 281of NIRC is conditioned upon the actual recovery
or collection of revenues, and no such revenue or income
was actually realized or recovered on any benefit accrued to
the government, since 2 government agencies were
involved. COA also impugned the propriety of the claim for
informer's reward based on inter- governmental violations.
Allowance of claims of the kind would not only place a
premium upon violations committed by government
agencies but also induce collusion among government
offices in order to obtain the informer's reward.
Commissioner of Internal Revenue sought reconsideration of
COA Decision. He was followed by Tirso Savellano and Mrs.
Evangelista, former Chief of the BIR Accounting Division

after (CSB) No. 89-0001-104 was directing the withholding of


salaries or any amount due them and to several BIR
officials/employees being held personally liable for the
disallowed amount of P11,397,924.75. COA denied
reconsideration. Consolidated Petitions:
ISSUE AND RULING:
WON the COA decision disallowing in audit payment
of informers reward to Savellano was valid- NO!
CIR: the approval by the Department of Finance of the claim
for informer's reward is conclusive upon the executive
agencies concerned, respondent COA included. Section 90 of
the Government Auditing Code of the Philippines; there were
actual cash collections of P109,941,644.17 from NCA and
PNOC for non-payment of withholding taxes on interest
earnings, which amount had accrued to the General Fund;
Section 316 (now 281) of the NIRC entitling an informer to a
reward for information leading to the collection of internal
revenue taxes is clear and needs no interpretation; NCA and
PNOC have separate personalities from the Bureau of
Internal Revenue and Government.
Tirso Savellano: Express statutory grant to BIR of the
power to allow or disallow claims for payment of tax
informer's reward is an implied statutory denial of the same
power to the COA, which would otherwise transform said
respondent into "a super tax authority" and "undermine and
dilute the substance and efficacy of the very entity created
and empowered by law to collect taxes and augment the
government's revenue collecting potentials". There was
"actual" collection of tax by the BIR from the NCA and PNOC
because while said agencies are GOCCs, they derive their
income from the exercise of proprietary functions, which
does not, in and by itself, constitute public funds.
Solicitor General: found COA's disallowance of the
informer's reward erroneous because government
corporations are subject to tax under the NIRC; having

12

TAXATION 1 | B2015
CASE DIGESTS

personalities distinct from the government. Section 281 of


the NIRC does not make any distinction among taxpayers
from whom taxes are eventually recovered; it simply
prescribes that for an informer to be entitled to the reward,
the information he furnishes should result in the recovery of
revenues. The possibility of collusion is not sufficient basis
for disallowance, since collusion cannot be assumed. The
determination made by the BIR should be binding upon COA
pursuant to the Government Auditing Code.
COA: CIR is not an aggrieved party adversely affected by
the assailed decisions. COA invokes its constitutionallyvested audit jurisdiction over all government agencies, to
which, it contends, the statutorily granted power of the
Secretary of Finance under Section 90, P.D. 1445 must yield.
Savellano is not entitled to the informer's reward because
there was no actual collection of revenues under the
benefit-to-the-government rule; and Savellano's alleged
information did not lead to the discovery of a fraud. The
payment of informer's reward as irregular, being predicated
upon violations committed by government agencies, and
would have the persons named in CSB No. 89- 0001-104 (c)
held liable for participation in illegal or irregular
disbursements of public funds by reason of their respective
duties.
SC:
Sec. 90 PD 1445. Payment of rewards. When a reward becomes payable
by authority of law for information given relative to any offense or for any
act done in connection with the apprehension of the offender, the reward
shall, in the absence of special provisions, be paid in such manner as shall
be prescribed by executive order. The final determination by the proper
administrative authority pursuant to law or any such order, as to whether
or not the persons concerned are entitled to any reward and the amount
thereof, shall be conclusive upon the executive agencies concerned as
regards the liability of the government.

The final determination by the Department of Finance,


through the recommendation of the BIR, of Savellano's
entitlement to the informer's reward is, under Section 90,

conclusive only upon the executive agencies concerned.


COA is not an executive agency. It is one of the 3
independent constitutional commissions. Specifically, it is
the constitutional agency vested with the "power, authority
and duty to examine, audit and settle all accounts
pertaining to the revenue and receipts of, and expenditures
or uses of funds and property owned or held in trust by the
government, or any of its subdivisions, agencies or
instrumentalities. . . ." To ensure the effective discharge of
its functions, it has been empowered, subject to the
limitations imposed by Article IX (D) of the 1987
Constitution, to define the scope of its audit and
examination, establish the techniques and methods required
therefore, and promulgate accounting and auditing rules
and regulations, including those for the prevention and
disallowance of irregular, unnecessary, excessive,
extravagant or unconscionable expenditures or uses of
government funds and properties.
The final determination made by the Finance Department
cannot bind respondent COA or foreclose its review thereof
in the exercise of its constitutional function and duty to
ensure that public funds are expended and used in
conformity with law. To hold otherwise would be to ignore
the clear mandate and the equally clear implications of
Section 3, Article IX (D)of the 1987 Constitution providing that:No law shall
be passed exempting any entity of the government of its subsidiary in any
guise whatever, or any investment of public funds, from the jurisdiction of
the Commission on Audit.

As held in Dingcong vs. Guingona, Jr., et al: Not only is the


COA vested with the power and authority, but it is also
charged with the duty, to examine, audit and settle all
accounts pertaining to the expenditures or uses of funds
owned by, or pertaining to, the Government or any of its
subdivisions, agencies, or instrumentalities. That authority
extends to the accounts of all persons respecting funds or
properties received or held by them in an accountable
capacity (Section 26, P.D. No. 1445). In, the exercise of its
jurisdiction, it determines whether or not the fiscal

13

TAXATION 1 | B2015
CASE DIGESTS

responsibility that rests directly with the head of the


government agency has been properly and effectively
discharged and whether or not there has been loss or
wastage of government resources. It is also empowered to
review and evaluate contracts. And, after an audit has been
made, its auditors issue a certificate of settlement to each
officer whose account has been audited and settled in whole
or in part, stating the balances found due thereon and
certified, and the charges or differences arising from the
settlement by reason of disallowances, charges or
suspensions
This is not to say, however, that the disallowance in audit by
respondent COA is in itself final. The same may be set aside
and nullified by the Cout, if done with grave abuse of
discretion.
The informer's reward granted to petitioner Savellano is
based on Section 316 (now 281) of the National Internal
Revenue Code. It reads:
Sec. 281. Informers reward to persons instrumental in the discovery of
violation of the National Internal Revenue Code and in the discovery and
seizure of smuggled goods. (1) For violation of the National Internal
Revenue Code. Any person except an internal revenue official or employee,
or other public official, or his relative within the sixth grade of
consanguinity, who voluntarily gives definite and sworn information, not yet
in the possession of the Bureau of Internal Revenue, leading to the
discovery of frauds upon internal revenue laws or violation of any of the
provisions thereof, thereby resulting in the recovery of revenues,
surcharges and fees and/or the conviction of the guilty party and/or
imposition of any fine or penalty, shall be rewarded in the sum equivalent
to fifteen per centum of the revenues, surcharges or fees recovered and/or
fine or penalty imposed and collected. The same amount of reward shall
also be given to an informer where the offender has offered to compromise
the violation of law committed by him and his offer has been accepted by
the Commissioner and in such a case, the fifteen per centum reward fixed
herein shall be based on the amount agreed upon in the compromise and
collected from the offender; Provided, That should no revenues, surcharges
or fees be actually recovered or collected, such person shall not be entitled
to a reward: Provided, further, That the information mentioned herein shall
not refer to a case already pending or previously investigated or examined
by the Commissioner or any of his deputies, agents or examiners, or the
Secretary of Finance or any of his deputies or agents: Provided, finally, That

the reward provided herein shall be paid under the regulations issued by
the Commissioner of Internal Revenue with the approval of the Secretary of
Finance.

COA's disallowance of the informer's reward on the ground


that there was actually no revenue realized or recovered as
2 government agencies were involved overlooks the fact
that NCA and PNOC, possess legal personalities separate
and distinct from the Philippine government. Although both
are GOCCs, they perform proprietary functions. Their
revenues do not automatically devolve to the general
coffers of the government. Unless transferred to the
Philippine government through the vehicle of taxation, no
part of their revenues is available for appropriation by the
Legislature for expenditure in government projects; such
revenues remain said agencies' in their entirety, to be
applied to and expended for their own exclusive purpose.
When revenues are subjected to tax, the portion thereof
corresponding to such tax becomes, in its own, revenue for
the government accruing to the General Fund.
That the informer's reward was sought and given in relation
to tax delinquencies of government agencies provides no
reason for disallowance. The law on the matter makes no
distinction whatsoever between delinquent taxpayers in this
regard, whether private persons or corporations, or public or
quasi-public agencies, it being sufficient for its operation
that the person or entity concerned is subject to, and
violated, revenue laws, and the informer's report thereof
resulted in the recovery of revenues. It is elementary that
where the law does not distinguish, none must be made. Ubi
lex non distinguit nec nos distinguere debemos.
Also, mere possibility of collusion to obtain the informer's
reward is not sufficient ground for disallowance. Collusion
cannot be presumed.Here, there is no showing of collusion
between Savellano and any official or employee of the BIR
or the Department of Finance. Neither is there any evidence
to overcome the presumption of regularity enjoyed by the
official acts of the BIR and the Department of Finance in

14

TAXATION 1 | B2015
CASE DIGESTS

approving the claim of petitioner Savellano for informer's


reward.
The delinquencies of these agencies are not condoned,
much less rewarded. It is the person whose information led
to the discovery of their transgressions who is being
rewarded. Although this results in a reduction in the amount
of revenues actually received, the net effect is that the
government still gains from the remaining amount paid,
which otherwise would have been lost to it.
DISPOSITIVE: consolidated
Decisions of COA set aside.

petitions

are

DOCTRINE:
Restated: Payment of a money judgment regarding a case
as per agreement of the parties constitutes taxable passive
income.
Verbatim, from the case: The rest of the income earned by
way of money judgment per agreement of the parties
constitutes taxable income [passive income] to the
Spouses Ramnani subject to the income tax under Section
21(f) of the NIRC, in the same manner as citizens of the
Philippines.

GRANTED.

Ramnani v. Commissioner
of Internal Revenue
September 13, 1996
Ernesto D. Acosta
Paolo Q. Bernardo

SUMMARY: Spouses Ramnani were parties in a case


decided by the SC. The SC decided in favor of the spouses
and against Choithram C. Ramnani, Moti C. Ramnani,
Nirmla V. Ramnani and Ortigas and Co. Ltd. Partnership.
Thereafter, both the Spouses and the losing parties
entered into an agreement which set the money value of
the judgment provisionally at P65 million.
The Commissioner of Internal Revenue thereafter issued a
BIR ruling. She informed the losing parties, that as payors
of the P65 Million, they are constituted as the withholding
agents of the 30% final income tax on the whole P65
million.
The Spouses Ramnani then filed a petition for review with
the CTA to set aside the questioned BIR ruling.

FACTS:
Spouses Ishwar Jethmal Ramnani and Sonya Jethmal
Ramnani (Spouses Ramnani) were parties in a case decided
by the Supreme Court (SC) on May 7, 1991, The SC in this
case decided in favor of the spouses and against Choithram
C. Ramnani, Moti C. Ramnani, Nirmla V. Ramnani and Ortigas
and Co. Ltd. Partnership (losing parties).
The Motion for Reconsideration of the aforequoted decision
filed by the losing parties was subsequently denied by the
SC.
The Regional Trial Court of Pasay was afterwards designated
by the SC to execute the judgment. It issued an Order of
partial writ of execution:
a) the rental income of the properties and
improvements from 1967 up to May 31, 1992 in
the amount of P24,879,365.00; and
b) the 1985 appraised value of the properties in the
amount of P22,364,000.00; P500,000.00 moral
damages; P200,000.00 exemplary damages and 10%
of said amounts as attorney's fees, plus 6% legal
interest on the totality of the amounts from the time
the judgment became final until fully paid.
On July 19, 1993, the parties, pursuant to the SC case
aforecited, entered into a tripartite agreement setting
forth the following terms:

15

TAXATION 1 | B2015
CASE DIGESTS

"2. To expeditiously terminate said proceedings, the


parties (excluding Moti and Nirmla Ramnani) hereby
set the money value of the said Judgment,
including the other monetary awards due Sps.
Ishwar provisionally at P65 Million. The final and
total monetary awards to Sps. Ishwar could be more
than P65 Million. It could also be less than said
amount.
Consequently, without waiting for the Court's
determination aforesaid, parties agree to suspend
hearings on valuation and proceedings in execution
of the judgment in G.R. No. 85494 and G.R. No.
85496.
Thereafter, Commissioner of Internal Revenue, Liwayway
Vinzons-Chato, issued a BIR ruling. She informed the losing
parties that as payors of the P65 Million, they are
constituted as the withholding agents of the 30% final
income tax provided in Section 22(b), in relation to Sections
50(a) and 51(a) of the Tax Code.
The amount to be withheld is Twenty Million One
Hundred Fifty Thousand (P20,150,000.00) Pesos,
broken down as follows:
30% final income tax on
P65,000,000.00 P19,500,000.00
1% documentary stamp tax on
P65,000,000.00 650,000.00

Total P20,150,000.00
===========
For the Commissioner, Section 22(b) of the Tax Code is the
applicable provision of taxing the receipt of the income by
non-resident American citizen, such as rents, casual gains,
profits, and income. Inasmuch as the compromise
settlement arose from a money judgment involving
ownership over real property, the income is taxable in the
Philippines notwithstanding the fact that the recipients
thereof, Spouses Ramnani, are American citizens, as
provided in Article 7(1) of the RP-US Tsx Treaty where the

location of the real property is the situs of income taxation


and not the residence of the alienator.
The Spouses Ramnani did not agree with BIR Ruling so in a
letter they requested the Commissioner to reconsider the
opinions, arguing that:
They are resident aliens contrary to the ruling's
declaration that they are non-resident aliens not
doing business in the Philippines;
The money judgment rendered in their favor is the
nature of a mere return of capital/investment and
hence must not be subject to the income tax.
In an effort to put more teeth to this aforequoted BIR ruling,
the Commissioner served a Warrant of Garnishment to the
Equitable Banking Corporation, the drawee bank, to distrain
and garnish the cheques issued in favor of Spouses Ramnani
in the amounts of P10,000,000.00 and P15,000,000.00 to
cover the payment of the final income tax of
P17,340,000.00.
The Spouses Ramnani then filed a petition for review with
the Court of Tax Appeals. They are asking the CTA to set
aside the questioned BIR ruling declaring them to be nonresident aliens not doing business in the Philippines and
subjecting the money judgment, excluding the award for
damages, to the income tax.
ISSUES: (1) WON petitioners are resident aliens; and
[Relevant] (2) WON the money judgment issued in their
favor are subject to income tax.
RULING: 1. Yes; 2. Yes.
RATIO:
1. We agree with the Spouses Ramnani that they are a
resident alien for income tax purposes.
a. Section 20(f) of the National Internal Revenue
Code (NIRC) defines a resident alien as an

16

TAXATION 1 | B2015
CASE DIGESTS

"individual whose residence is within Philippines


and who is not a citizen thereof"
i. Evidence adduced during the trial convinced
this Court that Ishwar Ramnani's stay in the
Philippines can no longer be considered as
transient.
b. Section 5 of Revenue Regulations No. 2
delineates the distinction between a person who
can be considered a resident and one who is not
by the following description:
i. "SEC. 5. Definition. A 'non-resident alien
individual' means an individual
ii. (a) Whose residence is not within the
philippines; and
iii. (b) Who is not a citizen of the Philippines.
c. An alien actually present in the Philippines who is
not a mere transient or sojourner is a resident of
the Philippines for purposes of the income tax.
Whether he is a transient or not is determined by
his intentions with regard to the length and
nature of his stay.
i. A mere floating intention indefinite as to time,
to return to another country is not sufficient to
constitute him a transient. If he lives in the
Philippines and has no definite intention as to
stay, he is a resident.
ii. One who comes to the Philippines for a
definite purpose which in its nature may be
promptly accomplished is a transient.
iii. But if his purpose is of such a nature that an
extended stay may be necessary for its
accomplishment, and to that end the alien
makes
his home
temporarily
in the
Philippines, he becomes a resident though it
may be his intention at all times to return to
his domicile abroad when the purpose for
which he came has been consummated or
abandoned."
d. It can be gleaned from the above description that
the law on income Taxation provides a more

liberal interpretation of who may be considered a


resident alien.
e. The establishment of a home even temporarily
here in the Philippines for the accomplishment of
a purpose even if he has the intention to return to
his domicile abroad categorizes an individual as a
resident.
f. There is no doubt that Ishwar Ramnani is an
American citizen who frequently comes to the
Philippines for the most part of the year to
oversee his various investments as shown by his
passport entries.
g.
The then Commissioner of Immigration even
approved the change of his status of admission
from temporary visitor to
immigrant/resident alien under Section 13(e) of the
Philippine Immigration Act.
2. It is clear from the Order of Writ of Execution that
only the amount of P24,879,265.00 represents rental
income from the lease of the subject properties and
improvements of Spouses Ramnani.
a. Spouses Ramnani admitted the receipt of rental
income amounting to P24,879,265.00.
i. Rental income is subject to 5% withholding
tax. Thus, the amount of P24,879,265.00,
representing rental income, should be subject
to the 5% withholding tax pursuant to Section
50(b) of the NIRC, in relation to Section 1(c) of
Revenue Regulations No. 6-85, as amended.
ii. From the facts presented by the SC, it has
been established that in 1966 the Spouses
have invested US$150,000.00 or about
P600,000.00 (US$150,000.00 x P4.00/US$1.00
in 1966) in real estate in the Philippines.
The amount of P600,000.00, representing
the capital Invested in the real estate
business in the Philippines in 1966, formed
part of the money judgment when the
court declared and awarded the appraised

17

TAXATION 1 | B2015
CASE DIGESTS

b.

c.

d.

e.

value of the properties and improvements


as
of
1985
in
the
amount
of
P22,364,000.00.
Thus,
the
capital
investment
of
P600,000.00 (US$150,000.00) should NOT
therefore form part of the taxable base for
income tax purposes since this is not
income but a mere return on capital.
iii. THUS, The 5% withholding tax on the rental
income of P24,879,265.00 or the amount of
P1,243,963.25 should be withheld by the
losing parties, as payors of the Spouses.
The CTA agrees that the moral and exemplary
damages as well as the attorney's fees are NOT
subject
to
income
tax.
However,
the
Commissioner failed to recognize that the capital
investment of P600,000.00 should likewise be
excluded from the computation of the gross
income for 1993.
The rest of the income earned by way of
money judgment per agreement of the
parties constitutes taxable income to the
Spouses Ramnani subject to the income tax
under Section 21(f) of the NIRC, in the same
manner as citizens of the Philippines.
As a resident alien, the Spouses are also entitled
to deductions and personal and additional
exemptions. Furthermore, as a resident alien,
Ramnani is required to file an income tax return
declaring therein the income awarded to him by
the
Court's
judgment,
amounting
to
P57,200,000.00 in the year he received such
income. The amount of P57,200,000.00 less
allowable deductions, either itemized or optional
standard deductions (40%), personal and
additional exemptions if any, shall be the net
taxable income subject to the income tax under
Section 21(f) of the NIRC.
The income tax due and payable shall be net of
the creditable withholding tax on rental income

amounting to P1,243,963.25 and the income tax


paid
on April
15, 1994 amounting
to
P3,823,021.22.
f. Based on the above discussion and in reference
to the amounts awarded to herein petitioner by
the Regional Trial Court of Pasay, the income tax
liability of the Spouses Ramnani is computed
herein below as follows:
i. Total Money Judgment (1993) P65,000,000.00
ii. Less:
1) capital investment P600,000.00
2) moral damages 500,000.00
3) exemplary damages 200,000.00
4) attorney's fees 6,500,000.007,800,000.00

Gross Income P57,200,000.00
Less: 40% Optional
Standard Deductions 22,880,000.00

Net Income P34,320,000.00Less: Personal Exemptions (RA


7167) 18,000.00

Net Taxable Income P34,302,000.00


===========
Income Tax Due P10,247,200.00
Less: Income Tax paid on 4/15194 3,823,021.22
5% Withholding tax onrentals 1,243,963.25

Income Tax Payable P5,180,215.53


===========
DISPOSITIVE:The BIR Rulings are SET ASIDE. Spouses
Ramnani are considered a resident alien taxable In
the same manner as a resident citizen under Section
21 (f) of the NIRC. The losing parties
are hereby ORDERED TO WITHHOLD the amount of
P1,243,968.25 representing the 5% withholding tax
on the rental of P24,879,365.00. Spouses Ramnani
are also hereby ORDERED to pay additional income

18

TAXATION 1 | B2015
CASE DIGESTS

tax as in the amount of P5,180,215.53 plus interest


computed from promulgation of decision
until fully paid.
DISSENTING OPINION:
CONCURRING OPINION:

ZAMORA vs. CIR and CTA


May 31, 1963
Paredes, J.
Ron

SUMMARY:
1. Zamora is contesting the deficiency income tax
assessed by the CIR.
2. The CIR disallowed half of the promotion expenses
which he deducted in computing his net income.
3. SC held that the disallowance was proper being the
promotion expenses being claimed was not used
exclusively for business purposes but also for personal
purposes and the promotion expense was not
supported by receipts.
DOCTRINE:
1. In computing net income, there shall be allowed as
deductions all the ordinary and necessary expenses
paid or incurred during the taxable year, in carrying on
any trade or business (Vol. 4, Mertens, Law of Federal
Income Taxation, sec. 25.03, p. 307).
(1) Since promotion expenses constitute one of the
deductions in conducting a business, same must testify
these requirements. Claim for the deduction of
promotion expenses or entertainment expenses must
also be substantiated or supported by record showing
in detail the amount and nature of the expenses
incurred (N.H. Van Socklan, Jr. v. Comm. of Int. Rev.; 33
BTA 544).
(2) Representation expenses fall under the category of

business expenses which are allowable deductions from


gross income, if they meet the following conditions:
i. that to be deductible, said business expenses must
be ordinary and necessary expenses paid or
incurred in carrying on any trade or business;
ii. that those expenses must also meet the further test
of reasonableness in amount;
iii. that when some of the representation expenses
claimed by the taxpayer were evidenced by
vouchers or chits, but others were without vouchers
or chits, documents or supporting papers;
iv. that there is no more than oral proof to the effect
that payments have been made for representation
expenses allegedly made by the taxpayer and
about the general nature of such alleged expenses;
v. that accordingly, it is not possible to determine the
actual amount covered by supporting papers and
the amount without supporting papers, the court
should determine from all available data, the
amount properly deductible as representation
expenses..
FACTS:
1. Mariano Zamora, owner of the Bay View Hotel and
Farmacia Zamora, Manila, filed his income tax returns
the years 1951 and 1952.
2. The CIR found that he failed to file his return of the
capital gains derived from the sale of certain real
properties and claimed deductions which were not
allowable.
3. The collector required him to pay the sums of
P43,758.50 and P7,625.00, as deficiency income tax for
the years 1951 and 1952.
4. Zamora filed case in CTA questioning the deficiency
income tax.
5. CTA modified the decision appealed from and ordered
Zamora to pay the reduced total sum of P30,258.00
(P22,980.00 and P7,278.00, as deficiency income tax for
the years 1951 and 1952, respectively)

19

TAXATION 1 | B2015
CASE DIGESTS

Zamora contention:
1. CTA erred in disallowing P10,478.50 as promotion
expenses incurred by his wife for the promotion of the
Bay View Hotel and Farmacia Zamora.
a. Zamora is claiming that his wife incurred P20,957.00
as promotion expenses and the whole amount should
be allowed as deductions and not only half of it or
P10,478.50.
b. The P20,957.00 was allegedly spent by Mrs.
Esperanza A. Zamora (wife of Mariano), during her
travel to Japan and the United States to purchase
machinery for a new Tiki-Tiki plant, and to observe
hotel management in modern hotels.
2. CTA erred in disallowing 3-% per annum as the rate of
depreciation of the Bay View Hotel Building but only 2-
%.
ISSUE/S:
1. WON the disallowance of a part the promotion expenses
proper?
2. WON the proper rate of depreciation is 2-%?
RATIO:
1. YES, as a general rule, in computing net income,
promotion expense is a deductible item, being part of
the ordinary and necessary expenses in carrying on any
trade or business. Provided, promotion expenses or
entertainment expenses must also be substantiated or
supported by record showing in detail the amount and
nature of the expenses incurred.
2. YES, the proper rate of depreciation is 2-%.
RULING:
2. In computing net income, there shall be allowed as
deductions all the ordinary and necessary expenses paid
or incurred during the taxable year, in carrying on any
trade or business (Vol. 4, Mertens, Law of Federal Income
Taxation, sec. 25.03, p. 307).
(1) Since promotion expenses constitute one of the
deductions in conducting a business, same must testify

these requirements. Claim for the deduction of


promotion expenses or entertainment expenses must
also be substantiated or supported by record showing in
detail the amount and nature of the expenses incurred
(N.H. Van Socklan, Jr. v. Comm. of Int. Rev.; 33 BTA 544).
(2) Not all of Mrs. Zamoras expenses came under the
category of ordinary and necessary expenses; part
thereof constituted her personal expenses.
i. She went abroad on a combined medical and business
trip.
ii. No receipt whatsoever, were submitted to explain the
alleged business expenses, or proof of the connection
which said expenses had to the business or the
reasonableness of the said amount of P20,957.00.
iii. There having been no means by which to ascertain
which expense was incurred by her in connection with
the business of Mariano Zamora and which was
incurred for her personal benefit, the Collector and the
CTA in their decisions, considered 50% of the said
amount of P20,957.00 as business expenses and the
other 50%, as her personal expenses.
(3) While in situations like the present, absolute certainty is
usually no possible, the CTA should make as close an
approximation as it can, bearing heavily, if it chooses,
upon the taxpayer whose inexactness is of his own
making.
(4) Representation expenses fall under the category of
business expenses which are allowable deductions from
gross income, if they meet the following conditions:
i. that to be deductible, said business expenses must
be ordinary and necessary expenses paid or incurred
in carrying on any trade or business;
ii. that those expenses must also meet the further test
of reasonableness in amount;
iii. that when some of the representation expenses
claimed by the taxpayer were evidenced by vouchers
or chits, but others were without vouchers or chits,
documents or supporting papers;
iv. that there is no more than oral proof to the effect
that payments have been made for representation

20

TAXATION 1 | B2015
CASE DIGESTS

expenses allegedly made by the taxpayer and about


the general nature of such alleged expenses;
v. that accordingly, it is not possible to determine the
actual amount covered by supporting papers and the
amount without supporting papers, the court should
determine from all available data, the amount
properly deductible as representation expenses.
2. Normally, an average hotel building is estimated to have
a useful life of 50 years.
But inasmuch as the useful life of the building for
business purposes depends to a large extent on the:
i. suitability of the structure to its use and location,
ii. its architectural quality,
iii. the rate of change in population,
iv. the shifting of land values,
v. as well as the extent and maintenance and
rehabilitation.
vi. It is allowed a depreciation rate of 2-%
corresponding to a normal useful life of only 40 years
(1955 PH Federal Taxes, Par 14 160-K).
Consequently, the stand of Zamora cannot be sustained.
DISPOSITIVE: IN VIEW HEREOF, the petition in each of the
above-entitled cases is dismissed, and the decision
appealed from is affirmed, without special pronouncement
as to costs.

ESSO STANDARD v. CIR


July 7, 1989
Cruz, J.
Denn

SUMMARY: ESSO claims for refund of its overpaid income


taxes for the years 1959 and 1960. It says that the margin
fees it paid Central Bank on its profit remittances to its New
York head office must be considered as an ordinary and
necessary business expense, deductible from its gross

income. The CIR disagreed and said that the margin fees are
not allowed as deductible business expenses. Upon appeal,
the CTA agreed with the CIR. The SC agreed with CIR and
CTA and held that margin fees are not ordinary and
necessary business expenses deductible from gross income.
DOCTRINE: When a taxpayer claims a deduction, he must
point to some specific provision of the statute in which that
deduction is authorized and must be able to prove that he is
entitled to the deduction which the law allows. An item of
expenditure, in order to be deductible under Section 30(a)
(1) of NIRC must fall squarely within its language.
STATUTORY TEST OF DEDUCTIBILITY: To be deductible as a
business expense, 3 conditions are imposed:
(1) The expense must be ordinary and necessary
(2) It must be paid or incurred within the taxable year,
and
(3) It must be paid or incurred in carrying on a trade or
business.
In addition, not only must the taxpayer meet the business
test, he must substantially prove by evidence or records the
deductions claimed under the law, otherwise, the same will
be disallowed. The mere allegation of the taxpayer that an
item of expense is ordinary and necessary does not justify
its deduction.
FACTS: This case involves 2 CTA cases wherein petitioner
Esso Standard Eastern Inc.s claims for refund of overpaid
income taxes were denied by the CTA.
- CTA Case 1251: ESSO deducted from its gross
income (GI) for 1959, as part of its ordinary and
necessary business expenses, the amount it had
spent for drilling and exploration of its petroleum
concessions. The Commissioner of Internal Revenue
disallowed this claim on the ground that the
expenses should be capitalized and might be written
off as a loss only when a dry hole should result.

21

TAXATION 1 | B2015
CASE DIGESTS

In its amended return, ESSO asked for a


refund of P323,729.00 for its abandonment as
dry holes of several of its oil wells. It also
claimed as ordinary and necessary expenses
P340,822.04 representing margin fees it had
paid to the CB on its profit remittances to its
New York head office. CIR granted a tax credit
of
only
P221,033.00,
disallowing
the
deduction for margin fees.
CTA Case 1558: CIR assessed ESSO a deficiency
income tax of P367,994.00 for year 1960, plus 18%
interest of P66,238.92. Total = P434,232.92. This
deficiency arose from the disallowance of the margin
fees of P1,226,647.72 paid by ESSO to CB on its
profit remittances.
o ESSO settled the deficiency assessment by
applying the tax credit mentioned above and
paying under protest the additional amount. It
later claimed the refund of its alleged
overpayment on the interest on its deficiency
income tax.
ESSO: the 18% interest should have
been imposed NOT on the total
deficiency but only on the difference
between the total deficiency and its
tax credit.
CIR: denied ESSOs claim. 18% must
be on the entire amount of the
deficiency tax.
o CIR: denied ESSOs claim for refund of
overpayment of its 1959 and 1960 income
taxes. Margin fees paid to CB could not be
considered taxes or allowed as deductible
business expenses.
o ESSO appealed to CTA and sought refund of
overpayment, claiming that margin fees were
deductible from GI either as (1) tax or as (2)
an ordinary and necessary business expense.
Even if the margin fees were not deductible,
o

there was still an overpayment representing


the excess interest.
CTA denied ESSOs claim for refund
representing margin fees BUT sustained its
claim for excess interest.
The SC upheld the CTAs decision re:
excess interest.
ESSO appealed the CTA decision
denying its claims for refund of the
margin fees.

ISSUES:
1. WON RA 20091 is a police power measure or a
revenue measure (If it is a revenue measure, then
the margin fees paid by ESSO should be deductible
from its GI under Sec. 39 of NIRC) police power!
2. If margin fees are not taxes, WON they are necessary
and ordinary business expenses, deductible from GI
NO!
RULING:
1. RA 2009 is a police power measure. The margin fee
was imposed by the State not in the exercise of its
power of taxation, but of its police power. Margin fees
are not taxes.
2. Margin fees are not necessary and ordinary business
expenses deductible form GI.
RATIO:
1. ESSO: Margin fees are TAXES. As seen from the
background and legislative history of the Margin Fee
Law, RA 2009 was nothing less than a revival of the
17% excise tax on foreign exchange imposed by RA
601. This was a revenue measure proposed by Pres.
Carlos P. Garcia to balance the budget for 1959-1960.
The measure was one of the major sources of

RA 2009: An Act to Authorize the Central Bank of the Philippines to


Establish a Margin Over Banks Selling Rates of Foreign Exchange

22

TAXATION 1 | B2015
CASE DIGESTS

revenue used to finance the ordinary operating


expenditures of the government.
- CTA as cited by SC: Only in extremely doubtful
matters of interpretation does legislative history of
an act of Congress become important.
o There are at least 2 cases where the Court
has held that a margin fee is NOT A TAX but
an exaction designed to curb the excessive
demands upon our international reserve.
o Caltex Phil. v. Acting Commissioner of
Customs: A margin levy on foreign exchange
is a form of exchange control or restriction
designed
to
discourage
imports
and
encourage exports, and ultimately, curtail
any excessive demand upon international
reserve in order to stabilize the currency. By
its nature, the margin levy is part of the rate
of exchange as fixed by the government.
A tax is levied to provide revenue for
government operations, while the
proceeds of the margin fee are applied
to
strengthen
our
countrys
international reserve.
o Chamber of Agriculture v. CB: A tax is a levy
for the purpose of providing revenue fro
government operations, while the proceeds of
the 20% retention are applied to strengthen
the CBs international reserve.
SC CONCLUSION: The margin fee was imposed by the State
in the exercise of its police power and not its power of
taxation.
2. ESSO: if the margin fees are not taxes, they are
nevertheless ordinary and necessary business
expenses deductible from GI. The margin fees were
paid for ESSOs remittance as part of its profits to the
NY head office. Such remittance was an expenditure
necessary and proper for the conduct of its corporate
affairs.

SC: Sec. 30 (a) of NIRC is the applicable provision.

In Atlas Consolidated Mining v. CIR, the SC laid down


the rules on the deductibility of business expenses.
o When a taxpayer claims a deduction, he must
point to some specific provision of the statute
in which that deduction is authorized and
must be able to prove that he is entitled to
the deduction which the law allows.
o An item of expenditure, in order to be
deductible under Section 30(a)(1) of NIRC
must fall squarely within its language.
o STATUTORY TEST OF DEDUCTIBILITY: To be
deductible as a business expense, 3
conditions are imposed:
(4) The expense must be ordinary and
necessary
(5) It must be paid or incurred within the
taxable year, and
(6) It must be paid or incurred in carrying on a
trade or business.
o In addition, not only must the taxpayer meet
the business test, he must substantially prove
by evidence or records the deductions
claimed under the law, otherwise, the same
will be disallowed.
The mere allegation of the taxpayer
that an item of expense is ordinary and

Sec. 30. Deductions form gross income. In


computing net income, there shall be allowed as
deductions
(a) Expenses:
(1) In general All ordinary and necessary
expenses paid or incurred during the taxable year
in carrying on any trade or business xxx
(2) Expenses allowable to non-resident alien
individuals and foreign corporations In the case
of non-resident alien individual or a foreign
corporation, the expenses deductible are the
necessary expenses paid or incurred in carrying
on any business or trade conducted within the
Philippines exclusively.

23

TAXATION 1 | B2015
CASE DIGESTS

necessary
does
not
justify
its
deduction.
The terms ordinary and necessary have no
adequate or satisfactory definitions. There are,
however, certain guiding principles worthy of serious
consideration.
o Ordinarily, an expense will be considered as
necessary where the expenditure is
appropriate and helpful in the development of
the taxpayers business.
o It will be ordinary when it connotes a
payment which is normal in relation to the
business of the taxpayer and the surrounding
circumstances. The term ordinary does not
require that the payments be habitual or
normal in the sense that the taxpayer makes
them often. The payment may be unique or
non-recurring.
The right of deduction depends in each case on the
particular facts and the relation of the payment to
the type of business.
CTA did not err when it held that the margin fees are
NOT expenses in connection with the production or
earning of ESSOs incomes in the Phil. They were
expenses incurred in the disposition of said incomes;
expenses for the remittances of funds after they
have already been earned by ESSOs Philippine
branch.
Since the margin fees were incurred for the
remittance of funds to ESSOs head office in NY,
which is a separate and distinct income taxpayer
from the Philippine branch, for its disposal abroad, it
can never be said that the margin fees were
appropriate and helpful in the development of
ESSOs business in the Philippines exclusively or
were incurred for the purposes proper to the conduct
of the affairs of ESSOs Philippine branch exclusively
or for the purpose of realizing a profit or minimizing a
loss in the Philippines exclusively.

ESSO failed to show that the remittance to the head


office of part of its profits was made in furtherance of
its own trade or business. It merely presumed that all
corporate expenses are necessary and appropriate in
the absence of a showing that they are illegal or
ultra vires. Such assumption is wrong. The
paramount rule is that claims for deductions are a
matter of legislative grace and do not turn on mere
equitable considerations The taxpayer in every
instance has the burden of justifying the allowance of
any deduction claimed.
CONCLUSION: ESSO, having assumed an expense
properly attributable to its head office, cannot now
claim this as an ordinary and necessary expense paid
or incurred in carrying its own trade or business.

DISPOSITIVE: CTA decision denying refund affirmed.

CM HOSKINS v. CIR (DE LEON)

Gancayco vs. C.I.R.


April 20, 1961
Concepcion, J.
Francis

SUMMARY: Gancayco was arguing that his farm expenses


should be deducted from the computation of the net
income as an ordinary and necessary expense. SC held
that No evidence has been presented as to the nature of
the said "farming expenses" other than the bare
statement of Gancayco that they were spent for the
"development and cultivation of his property". No
specification has been made as to the actual amount spent
for purchase of tools, equipment or materials, or the

24

TAXATION 1 | B2015
CASE DIGESTS

amount spent for improvement. CIR claims that the entire


amount was spent exclusively for clearing and
developing the farm which were necessary to place it in a
productive state. It is not, therefore, an ordinary expense
but a capitol expenditure
DOCTRINE: An item of expenditure, in order to be
deductible
under
the
Tax
Code
as ordinary and necessary business
expenses,
must
fall squarely within the language of the statutory provision.
Such section is intended primarily, although not always
necessarily, to cover expenditures of a recurring nature
where the benefit derived from the payment is realized and
exhausted within the taxable year. Accordingly, if the result
of the expenditure is the acquisition of an asset which has
an economically useful life beyond the taxable year,
no deduction of such payment may be obtained under the
provisions of the statute. In such cases, deduction must be
obtained under the provisions of the statute which permit
deductions for amortization, depreciation, depletion or
loss.
FACTS: Santiago Gancayco filed his income tax return for
1949. 2 days later, CIR, Collector of Internal Revenue(CIR)
issued a notice advising him that his income tax liability for
1949 is P9,793.62, which he paid on May 15, 1950. CIR
wrote Gancayco stating that upon investigation, there was
still due a deficiency income tax for 1949 of P29,554.05.
Gancayco sought a reconsideration, which was partly
granted by reducing it to P16,860.31. Gancayco urged
another reconsideration but no action was taken on this
request.
CIR issued a warrant of distraint and levy against the
properties of Gancayco for the satisfaction of his deficiency
income tax liability. The municipal treasurer of issued a
notice of sale of said property at public auction. Gancayco
filed a petition to the CTA but the latter held he is liable for
ordered P16,860.31.

There are two 2 civil remedies for the collection of internal


revenue taxes: (a) by distraint of personal property and levy
upon real property; and (b) by judicial action. The first may
not be availed of except within 3 years after the "return is
due or has been made ..." Gancayco's income tax return for
1949 was filed on May 10, 1950 so the warrant of distraint
and levy issued on May 15, 1956, was illegal and void. The
judicial action in the Tax Code may be resorted to within 5
years from the date the return has been filed, if there has
been no assessment, or within 5 years from the date of the
assessment made within the statutory period. The CIR made
three 3 assessments: (a) the original assessment of
P9,793.62, on May 12, 1950; (b) the first deficiency income
tax assessment of May 14, 1951, for P29,554.05; and (c) the
amended deficiency income tax assessment of April 8,
1953, for P16,860.31.
ISSUES: Whether or not the
deductible from the net income

Farming

expenses

are

RATIO: No, they are capital expenditures, not ordinary and


necessary expenses
RULING:
Gancayco argues that the five-year period for the judicial
action should be counted from May 12, 1950, the date of the
original assessment, because the income tax for 1949, he
says, could have been collected from him since then.
SC: Said assessment was, however, not for the deficiency
income tax involved in this proceedings, but for P9,793.62,
which he paid.Hence, there never had been any cause for a
judicial action against him, and, per force, no statute of
limitations to speak of, in connection with said sum of
P9,793.62. The 2nd assessment cannot apply since it was
reconsidered on Gancayco's request. The last assessment is
what Gancayco contested in the amended petition filed by
him with the CTA. Thus, the five-year period should be
counted from April 8, 1953, so that the statute of limitations

25

TAXATION 1 | B2015
CASE DIGESTS

does not bar the present proceedings, instituted on April 12,


1956, if it is a judicial action.
Gancayco claims a deduction of 2 items, namely: (a) for
farming expenses, P27,459.00; and (b) for representation
expenses, P8,933.45.
SC: Section 30 of the Tax Code partly reads:
(a) Expenses:
(1) In General All the ordinary and necessary expenses
paid or incurred during the taxable year in carrying on any
trade or business, including a reasonable allowance for
salaries or other compensation for personal services actually
rendered; traveling expenses while away from home in the
pursuit of a trade or business; and rentals or other
payments required to be made as a condition to the
continued use or possession, for the purposes of the trade
or business, of property to which the taxpayer has not taken
or is not taking title or in which he has no equity.
On whether the farming expenses are such ordinary and
necessary expenses deductible from the net income it was
held that:
No evidence has been presented as to the nature of the
said "farming expenses" other than the bare statement of
Gancayco that they were spent for the "development and
cultivation of his property". No specification has been made
as to the actual amount spent for purchase of tools,
equipment or materials, or the amount spent for
improvement. CIR claims that the entire amount was spent
exclusively for clearing and developing the farm which
were necessary to place it in a productive state. It is not,
therefore, an ordinary expense but a capitol expenditure.
In computing net income, no deduction shall be allowed in
respect of any amount paid out for new buildings or for
permanent improvements, or betterments made to increase
the value of any property or estate.
Section 31 of the Tax Code provides that:

(a) General Rule In computing net income no deduction


shall in any case be allowed in respect of
(1) Personal, living, or family expenses;
(2) Any amount paid out for new buildings or for permanent
improvements,
or betterments made
to increase
the
value of any property or estate;
(3) Any amount expended in restoring property or in making
good the exhaustion thereof for which an allowance is or
has been made; or xxx
Expenses incident to the acquisition of property follow the
same rule as applied to payments made as direct
consideration for the property. Non-deductible expenses
include amounts paid in connection with geological
explorations,
development and
subdividing
of
real
estate; clearing and grading; restoration of soil, drilling
wells, architects's fees and similar types of expenditures.
See DOCTRINE
Disallowance of Gancayco's claim for representation
expenses is justified by the record, for, apart from the
absence of receipts, invoices or vouchers of the
expenditures in question, he could not specify the items
constituting the same, or when or on whom or on what they
were incurred.
DISPOSITIVE: Judgment of the CTA is AFFIRMED. Gancayco
is liable for deficiency tax of P16,860.31

Commissioner of Customs
vs Court of Tax Appeals
May 29, 1971
Makalintal, J.
Victor

SUMMARY: Philippine Acetylene imported a gas tank from


the United States. It was assessed by the Commissioner of

26

TAXATION 1 | B2015
CASE DIGESTS

Customs a special import tax. The company paid in protest.


Upon appeal to the CTA, the company was held to be
entitled to a refund. The SC reversed.

Upon appeal to the CTA, the CTA held that the company is
entitled to a refund. Thus, the commissioner appealed to the
SC.

DOCTRINE: Section 6 of RA 1394 is ambiguous. This being


the case, the statutory construction principle that tax
exemptions are held strictly against the taxpayer, and if
not expressly mentioned in the law must be within its
purview by clear legislative intent is applicable.

ISSUES:
WON Philippine Acetylene is an industry exempt from
special import tax under Section 6 of RA 13942 -> NO
WON the ambiguity of Section 6 should be construed in
favor of Philippine Acetylenes exemption -> NO (relevant)

FACTS: Philippine Acetylene Company (the company) is


engaged in the manufacture of oxygen, acetylene and
nitrogen, and the packaging of LPG in cylinders and tanks.
Sometime in 1957, the company imported from the United
States a custom-built LPG tank. It was then assessed by the
Commissioner of Customs (the commissioner) P 3,683 as
special import tax. The company paid in protest.

RULING: Philippine Acetylene is not exempt from the


special import tax

According to the manager of the company, the gas tank is


simply a large cylinder which is used as a container for LPG
obtained from the Caltex refinery in Batangas and
transported to the companys plant in Manila. The LPG does
not undergo any chemical change and is sold to consumers
in the same state, except that the gas is pumped into
smaller containers to be sold.

RATIO:
1. The SC upheld the contention of the commissioner, which
it quoted verbatim. It goes as follows: In the exempting
provisions of RA 1394, the exempted items are divided into
separate and distinct enumerations. The first group of
exempted industries refers exclusively to those falling under
the new and necessary industries as defined in Republic Act
No. 901. In the second, the term "industries" is classed
together with the terms miners, mining enterprises, planters
and farmers. ... If Congress really intended to give the term
"industries" its ordinary and general meaning and thus grant
tax exemption to all ventures and trades falling under the
said ordinary and general definition, it should have
eliminated the words "new and necessary industries' and
2

The tax provided for in section one of this Act shall not be
imposed against the importation into the Philippines of
machinery
and/or raw materials to be used by new and necessary
industries as
determined in accordance with Republic Act numbered Nine
Hundred
and One; ...; machinery, equipment, accessories and spare
parts, for
the use of industries, miners, mining enterprises planters
and
farmers; ...

27

TAXATION 1 | B2015
CASE DIGESTS

'mining enterprises" since these two ventures are already


covered by the term "industries" in its ordinary and general
meaning. On the other hand, the fact that the language of
the law specifically segregates new and necessary industries
under Republic Act No. 901 among those entitled to the tax
exemption, in effect, restricts the meaning and scope of the
word "industries.
2. Section 6 of RA 1394 is ambiguous. This being the case,
the statutory construction principle that tax exemptions are
held strictly against the taxpayer, and if not expressly
mentioned in the law must be within its purview by clear
legislative intent is applicable. As such, Philippine Acetylene
has to pay the special import tax.
DISPOSITIVE:
Acetylene.

CTA

reversed.

Costs

against

Philippine

GOODRICH
INTERNATIONAL RUBBER
CO. v. COLLECTOR OF
INTERNAL REVENUE
CTA Case No. 468
Luciano, Noel Christian O.
I am not sure of this is the correct CTA case.

SUMMARY: Goodrich has this policy of requiring its staff to


be members of athletic and social clubs for the purpose of
establishing contracts with prospective customers and to
entertain customers. For its taxable year 1952, Goodrich
declared as deductibles representation expenses. These
expenses are comprised of athletic and social clubs dues
and entertainment expenses. CIR disallowed the deduction
and assessed Goodrich with deficiency income tax.
Goodrich appealed.

DOCTRINE: With respect to club dues, it has been held


that where a corporation requires its officers to be
members of social or athletic clubs to promote its business,
and the club dues are paid by corporation, such dues are
DEDUCTIBLE as ordinary and necessary business expenses.
As regards the portion of the representation expenses
pertaining to entertainment expenses, it has been
sufficiently proven that said expenses were incurred to
entertain Goodrichs customers.
NOTE: This is an AMENDED DECISION; I cant find the
original decision as of the time of this writing
FACTS: I reconstituted the facts based from this 3-page
amended decision since I have not read the original
decision.
It appears that Goodrich required the members of its
staff to be members of athletic and social clubs for the
purpose of establishing contracts with prospective
customers and to entertain customers.
Goodrich claims that the expenses incurred from
these activities as REPRESENTATION EXPENSES. A portion of
these expenses
pertain to club dues and the rest to
entertainment expenses.
Goodrich declared these expenses for 1952
(amounting to P30,138.88) are deductible from its taxable
income. Hence, Goodrich deducted the said amount.
However, the CIR disallowed this deduction and
assessed Goodrich with a deficiency tax of P8,439.
JUNE 8, 1963 DECISION OF THE CTA:
1. The CTA reversed Goodrichs assessment for 1951
2. But affirmed the 1952 assessment
3. Thus, Goodrich was ordered to pay deficiency income tax
for 1952 (P8,439)
GOODRICHS
MOTION
FOR
RECONSIDERATION:
Goodrich filed an MR as regards the affirmance of the 1952
assessment. It contends that the finding is not supported by
evidence.

28

TAXATION 1 | B2015
CASE DIGESTS

CIR v. Palanca, Jr.

ISSUE: WON the said expenses are deductible.


RULING: YES!
HELD:
Remember that the REPRESENTATION
composed of:
a. Club dues
b. Entertainment expenses

EXPENSES

are

CLUB DUES ARE DEDUCTIBLE


With respect to club dues, it has been held that where a
corporation requires its officers to be members of social or
athletic clubs to promote its business, and the club dues are
paid by corporation, such dues are DEDUCTIBLE as ordinary
and necessary business expenses.
ENTERTAINMENT EXPENSES ARE ALSO DEDUCTIBLE
As regards the portion of the representation expenses
pertaining to entertainment expenses, it has been
sufficiently proven that said expenses were incurred to
entertain Goodrichs customers.
Goodrich, as a matter of policy, required its officers to be
members of social clubs, for purposes of establishing
contracts with prospective customers, particularly those
who own automobiles. The officers entertained Goodrichs
customers at these clubs and the expenses incurred were
reimbursed to the former.
It was also established that said expenses were incurred and
actually paid in 1952.
DISPOSITIVE: Dispositive portion of the June 8, 1963
decision is amended. Goodrich not liable for deficiency
income tax.

October 29, 1966


Regala, J.
Manzano: I only put the ruling with respect to the prescription
issue as to make this digest concise.

SUMMARY: Palanca Jr. received shares of stocks from his


deceased father and was assessed for estate taxes. He
paid his tax due but incurred interests thereon for
delinquent payment. He paid the delinquency interests.
The following year, he applied as deductions to his gross
income the delinquency interests he has paid.
For the second time he was assessed estate and
inheritance taxes on the same shares of stocks. Palanca
again paid interests for delinquency. Again, he applied the
delinquency interests as deductions on his gross income.
Since at that time he has already paid his income taxes, he
was asking for a refund of the excess taxes he has paid.
The issue here is whether interests on taxes for delinquent
payment may be considered as interests on indebtedness
as to be allowed as deduction under Section 30(b) (1) of
the Tax Code. The SC ruled, in this case, that although
taxes and indebtedness are the two different things,
interests paid on delinquency payments may be
considered as interests paid on indebtedness.
DOCTRINE: While the distinction between "taxes" and
"debts" was recognized in this jurisdiction, the variance in
their legal conception does not extend to the interests paid
on them, at least insofar as Section 30 (b) (1) of the
National Internal Revenue Code is concerned.
FACTS:
-Sometime in July, 1950: the late Don Carlos Palanca, Sr.
donated in favor of his son, Palanca Jr., 12,500 shares of
stock in La Tondea, Inc. For failure to file a return on the
donation Palanca Jr. was assessed the sums of P97,691.23,
P24,442.81 and P47,868.70 as gift tax, 25%

29

TAXATION 1 | B2015
CASE DIGESTS

surcharge and interest, respectively, which he paid on June


22, 1955.

P191,591.62, the petitioner paid the amount of P60,581.80


as interest for delinquency.

-March 1, 1956: the Palanca Jr. filed with the Bureau of


Internal
Revenue his income tax return for the calendar year 1955,
claiming,
a deduction for interest amounting to P9,706.45 and
reporting a taxable income of P65,982.12. On the basis of
this, he was assessed the sum of P21,052.91 as income tax
which he paid.

-August 12, 1958: the Palanca Jr. filed an amended income


tax return for the calendar year 1955, claiming, in addition
to the interest deduction of P9,076.45 appearing in his
original return, a deduction in the amount of P60,581.80,
representing interest on the estate and inheritance taxes on
the 12,500 shares of stock, thereby reporting a net taxable
income for 1955 in the amount of P5,400.32 and an income
tax due thereon in the sum of P428.00. Attached to this
amended return was a letter wherein Palanca Jr. requested
the refund of P20,624.01 which is the difference between
the amounts of P21,052.01 he paid as income tax under his
original return and of P428.00.
-CIR now seeks the reversal of the CTA's ruling. Specifically,
the issue with CTA's determination that the amount paid by
Palanca for interest on his delinquent estate and inheritance
tax is deductible from the gross income for that year under
Section 30 (b) (1) of the Revenue Code, and, that said
Palanca's claim for refund has not prescribed.

-November 10, 1956: the Palanca Jr. filed an amended return


for the calendar year 1955, claiming additional deduction in
the amount of P47,868.70 representing interest paid on
the donee's gift tax, thereby reporting a taxable net income
of P18,113.42 and a tax due thereon in the sum of
P3,167.00.
-The claim for deduction was based on the provisions of
Section 30(b)
(1) of the Tax Code, which authorizes the deduction from
gross income of interest paid within the taxable year
on indebtedness. A claim for the refund of alleged
overpaid income taxes for the year 1955 amounting to
P17,885.01, which is the difference between the amount of
P21,052.01 and of P3,167.00, was filed together with this
amended return. BIR denied the claim for refund.
-Meanwhile, the Bureau of Internal Revenue considered the
transfer of
12,500 shares of stock of La Tondea Inc. to be a transfer in
contemplation of death pursuant to Section 88(b) of the
National
Internal Revenue Code. Consequently, the CIR assessed
against the Palanca the sum of P191,591.62 as estate and
inheritance taxes on the transfer of said 12,500 shares of
stock. The amount of P17,002.74 paid by the Palanca Jr. as
gift tax, including interest and surcharge was applied to his
estate and inheritance tax liability. On the tax liability of

CIRs Argument: A tax is not an indebtedness. Debts are


due to the government in its corporate capacity, while taxes
are due to the government in its sovereign capacity. A debt
is a sum of money due upon contract express or implied or
one which is evidenced by a judgment. Taxes are imposts
levied by government for its support or some special
purpose which the government has recognized. In view of
this distinction, CIR submits that the deductibility of
"interest on indebtedness" from a person's income tax
under Section 30(b) (1) cannot extend to "interest on taxes.
ISUES:
1. WON an interest paid delinquent payment of
estate and inheritance tax can be considered
as an interest on indebtedness as to be
allowed as a deduction on Palancas gross
income? YES.

30

TAXATION 1 | B2015
CASE DIGESTS

While "taxes" and "debts" are distinguishable legal


concepts, in certain cases as in the case at bar, on account
of their nature, the distinction becomes inconsequential.
The term "debt" is properly used in a comprehensive sense
as
embracing not merely money due by contract, but whatever
one is
bound to render to another, either for contract or the
requirements of
the law. Where statutes impose a personal liability for a tax,
the tax becomes at least in a broad sense, a debt. Some
American authorities hold that Federal taxes are debts.
In our jurisdiction, the rule is settled that although taxes
already due have not, strictly speaking, the same concept
as debts, they are obligations that may be considered as
such.
In Commissioner of Internal Revenue vs. Prieto, the SC
explicitly announced that while the distinction between
"taxes" and "debts" was recognized in this jurisdiction, the
variance in their legal conception does not extend to the
interests paid on them, at least insofar as Section 30 (b) (1)
of the National Internal Revenue Code is concerned. Thus,
under the law, for interest to be deductible, it must be
shown that there be an indebtedness, that there should be
interest upon it, and that what is claimed as an interest
deduction should have been paid or accrued within the year.
It is here conceded that the interest paid by Palanca was in
consequence of the late payment of her donor's tax, and the
same was paid within the year it is sought to be deducted.
Section 30(b) (1) of the Tax Code reads:
Sec. 30. Deductions from gross income In computing net income
there shall be allowed as deductions
xxx xxx xxx
"Interest:
(1) In general. The amount of interest paid within the taxable year
on indebtedness, except on indebtedness incurred or continued to

purchase or carry obligations the interest upon which is exempt from


taxation as income under this Title.

The term "indebtedness" as used in the Tax Code of the US


containing similar provisions as in the above-quoted section
has been defined as the unconditional and legally
enforceable obligation for the payment of money. Within the
meaning of that definition, it is apparent that a tax may be
considered an indebtedness.
It follows that the interest paid by Palanca Jr. for the late
payment of his donor's tax is deductible from his income
under section 30 (b) of the Tax Code.
Of course, what was involved in the Prieto case was the
donor's tax while the present suit pertains to interest paid
on the estate and inheritance tax. This difference submits
no appreciable consequence to the rationale of this Court's
previous determination that interests on taxes should be
considered as interests on indebtedness within the meaning
of Section 30(b) (1) of the Tax Code. This interpretation upon
the said section was predicated on the congressional intent,
not on the nature of the tax for which the interest was paid.
2. WON the claim for refund has already
prescribed?
NO,
CIR's
contention
on
prescription is untenable.
DISPOSITIVE: SC affirmed the CTA and ruled in favor of
Palanca Jr.

CIR v. Vda. de Prieto


September 30, 1960
J. Gutierrez David
Jerome Marcelo

SUMMARY: Consuelo donated real property to her 4


children. She was delinquent in paying her corresponding

31

TAXATION 1 | B2015
CASE DIGESTS

donors tax, hence she had to pay an additional


P55,978.65. Thus she claimed this amount as a deduction
for her ITR for the same year. The CIR did not allow the
deduction. CTA: CIR decision reversed. SC: CTA affirmed.
DOCTRINE: For interest to be deductible from gross
income, it must be shown that there is an indebtedness,
that there should be interest upon it, and that what is
claimed as an interest deduction should have been paid or
accrued within the year.
The term "debt" for the purposes of tax deductions, is
properly used in a comprehensive sense as embracing not
merely money due by contract but whatever one is bound
to render to another, either by contract, or by law such as
taxes.
FACTS: December 4, 1945- the respondent Consuelo L. Vda.
de Prieto conveyed by way of gifts to her 4 children real
property with a total assessed value of P892,497.50. After
the filing of the gift tax returns, the Commissioner of Internal
Revenue appraised the real property donated for gift tax
purposes at P1,231,268.00, and assessed the total sum of
P117,706.50 as donor's gift tax, interest and compromises
due. Of the total sum of P117,706.50 paid by Consuelo on
April 29, 1954, the sum of P55,978.65 represents the total
interest on account of delinquency in paying on time. This
sum of P55,978.65 was claimed as deduction by Consuelo in
her income tax return for 1954.
Petitioner CIR, however, disallowed the claim and as a
consequence of such disallowance assessed Consuelo
P21,410.38 as deficiency income tax due on the aforesaid
P55,978.65, including interest up to March 31, 1957,
surcharge and compromise for the late payment. Consuelo
challenged the CIR ruling with the CTA.
CTA: CIR decision reversed.
valid.

Deduction of P55,978.65 is

Note: For interest to be deductible, it must be shown that


there is an indebtedness, that there should be interest upon

it, and that what is claimed as an interest deduction should


have been paid or accrued within the year. It is here
conceded that the interest paid by Consuelo was in
consequence of the late payment of her donor's tax, and the
same was paid within the year it is sought to be declared.
The only question to be determined, as stated by the
parties, is whether or not such interest was paid upon an
indebtedness within the contemplation of the Tax Code part
on deductions to gross income.
ISSUE and RULING: WON the P55,978.65, which is the
interest on account of delinquency in paying the donors tax,
can be claimed as deduction in the income tax return. YES
RATIO: The relevant law provides:

SEC. 30 Deductions from gross income. In computing net


income there shall be allowed as deductions
(b) Interest:
(1) In general. The amount of interest paid within the
taxable year on indebtedness, except on indebtedness incurred or
continued to purchase or carry obligations the interest upon which
is exempt from taxation as income under this Title.

The term "indebtedness" as used in the Tax Code of the US


containing similar provisions as in the above-quoted section
has been defined as an unconditional and legally
enforceable obligation for the payment of money. From that
definition, it is apparent that a tax may be considered an
indebtedness. As ruled by this court in Sambrano v. CTA:
Although taxes already due have not, strictly speaking, the
same concept as debts, they are, however, obligations that
may be considered as such.
The term "debt" is properly used in a comprehensive
sense as embracing not merely money due by contract but
whatever one is bound to render to another, either for
contract, or the requirement of the law. Where statute
imposes a personal liability for a tax, the tax becomes, at
least in a broad sense, a debt. A tax is a debt for which a
creditor's bill may be brought in a proper case.

32

TAXATION 1 | B2015
CASE DIGESTS

It follows that the interest paid by Consuelo for the late


payment of her donor's tax is deductible from her gross
income under section 30(b) above quoted.
The above conclusion finds support in the established
jurisprudence in the US after whose laws on Income Tax Law
ours has been patterned. In that jurisdiction, the uniform
ruling is that interest on taxes is interest on indebtedness
and is deductible.
To sustain the proposition that the interest payment in
question is not deductible for the purpose of computing
respondent's net income, petitioner CIR relies heavily on
section 80 of Revenue Regulation 2 promulgated by the
DOF, which provides that the word `taxes' means taxes
proper and no deductions should be allowed for amounts
representing interest, surcharge, or penalties incident to
delinquency. The court below, however, held section 80 as
inapplicable to this case because while it implements
sections 30(c) of the Tax Code governing deduction of taxes,
Consuelo seeks to come under section 30(b) of the same
Code providing for deduction of interest on indebtedness.
We find the lower court's ruling to be correct. Contrary to
petitioner's belief, the portion of section 80 of Revenue
Regulation 2 under consideration has been part and parcel
of the development to the law on deduction of taxes in the
US. Thus, Mertens in his treatise says: "Penalties are to be
distinguished from taxes and they are not deductible under
the heading of taxs." Interest on state taxes is not
deductible as taxes. This notwithstanding, courts in that
jurisdiction, however, have invariably held that interest on
deficiency taxes are deductible, not as taxes, but as interest.
Section 80 of Revenue Regulation 2, merely incorporated
the established application of the tax deduction statute in
the US, where deduction of "taxes" has always been limited
to taxes proper and has never included interest on
delinquent taxes, penalties and surcharges.
To give to the quoted portion of section 80 of our
Income Tax Regulations the meaning that the petitioner
gives it would run counter to the provision of section 30(b)
of the Tax Code and the construction given to it by US
courts. As already stated, section 80 implements only

section 30(c) of the Tax Code, or the provision allowing


deduction of taxes, while herein Consuelo seeks to be
allowed deduction under section 30(b), which provides for
deduction of interest on indebtedness.
In conclusion, we are of the opinion and so hold that
although interest payment for delinquent taxes is not
deductible as tax under Section 30(c) of the Tax Code and
section 80 of the Income Tax Regulations, the taxpayer is
not precluded thereby from claiming said interest payment
as deduction under section 30(b) of the same Code.
DISPOSITIVE: The decision sought to be reviewed is
affirmed, without pronouncement as to costs.

PAPER INDUSTRIES vs. CA


December 1, 1995
Feliciano, J.
Luisa
SUMMARY: Paper Industries Corporation of the Philippines
(PICOP) received from CIR two letters of assessment and
demand for deficiency transaction tax and documentary
and science stamp tax and eficiency income tax for 1977
PICOP protested the assessment of deficiency
transaction tax and documentary and science stamp taxes
and also the deficiency income tax assessment for 1977.
These werent formally acted upon but instead CIR issued
a warrant of distraint on personal property and a warrant of
levy on real property against PICOP in effect denying
PICOPs protests.
CTA modified CIR holding PICOP liable for the reduced
aggregate amount which represented 35% transaction tax,
documentary and science stamp tax and deficiency income
tax
CA reduced PICOPs liability but still held them liable.
SC held them liable but held that PICOP is entitled to
deductions for interest payments on loans.
DOCTRINE:
Section 30 of the 1977 Tax Code provides that

33

TAXATION 1 | B2015
CASE DIGESTS

interest payments on loans incurred by a taxpayer are


allowed by the NIRC as deductions against the taxpayers
gross income. Thus, the general rule is that interest
expenses are deductible against gross income and this
certainly includes interest paid under loans incurred in
connection with the carrying on of the business of the
taxpayer.
FACTS:
Paper Industries Corporation of the Philippines
(PICOP) is a Philippine corporation registered with the Board
of Investments (BOI) as a preferred pioneer enterprise with
respect to its integrated pulp and paper mill and as a
preferred non-pioneer enterprise with respect to its
integrated plywood and veneer mills.
PICOP received from CIR two letters of assessment
and demand for the following:
- Deficiency transaction tax and documentary and
science stamp tax
- Deficiency income tax for 1977
The two amounted to P88,763,255.00.
PICOP protested the assessment of deficiency
transaction tax and documentary and science stamp taxes
and also the deficiency income tax assessment for 1977.
These werent formally acted upon but instead CIR issued a
warrant of distraint on personal property and a warrant of
levy on real property against PICOP in effect denying
PICOPs protests.
PICOP appealed to CTA. CTA modified CIR holding PICOP
liable for the reduced aggregate amount of P20,133,762.33
which represented 35% transaction tax, documentary and
science stamp tax and deficiency income tax
Both parties appealed. CA reduced PICOPs liability to
P6,338,354.70 which represented 35% transaction tax in a
reduced rate, 20% interest per annum on the deficiency
income tax for a period of 3 years and a surcharge of 10%.
CA exempted PICOP from payment of documentary and
science stamp tax and compromise penalty.
ISSUES:
1. WON PICOP is liable for:

a. 35% transaction tax


b. Interest and surcharge on unpaid transaction
tax
c. Documentary and science stamp tax
2. WON PICOP is entitled to deductions against income
of:
a. (RELEVANT) Interest payments on loans for
the purchase of machinery and equipment
b. Net operating losses incurred by the Rustan
Pulp and Paper Mills Inc
c. Certain claimed financial guarantee expenses
RULING:
(1) Picop is liable for the thirty-five percent (35%)
transaction tax in the amount of P3,578,543.51.
(2) Picop is not liable for interest and surcharge on unpaid
transaction tax.
(3) Picop is exempt from payment of documentary and
science stamp taxes in the amount of P300,000.00 and the
compromise penalty of P300.00.
(4) Picop is entitled to its claimed deduction of
P42,840,131.00 for interest payments on loans for, among
other things, the purchase of machinery and equipment.
(5) Picop's claimed deduction in the amount of
P44,196,106.00 forthe operating losses previously incurred
by RPPM, is disallowed for lack of merit.
(6) Picop's claimed deduction for certain financial guarantee
expenses in the amount P1,237,421.00 is disallowed for
failure adequately to prove such expenses.
(7) Picop has understated its sales by P2,391,644.00 and
overstated its cost of sales by P604,018.00, for 1977.
(8) Picop is liable for the corporate development tax of five
percent (5%) of its adjusted net income for 1977 in the
amount of P2,434,367.75.
RATIO:
WON PICOP IS LIABLE FOR 35% TRANSACTION TAX
yes
Picop issued commercial paper consisting of serially
numbered promissory notes with the total face value of
P229,864,000.00 and a maturity period of one (1) year.
These promissory notes were purchased by various

34

TAXATION 1 | B2015
CASE DIGESTS

commercial banks and financial institutions. On these


promissory notes, Picop paid interest in the aggregate
amount of P45,771,849.00.
In respect of these interest payments, the CIR
required Picop to pay the thirty-five percent (35%)
transaction tax. Based on PD 1154 which stated:
Sec. 195-C. Tax on certain interest. There shall be
levied, assessed, collected and paid on every commercial
paper issued in the primary market as principal instrument,
a transaction tax equivalent to thirty-five percent (35%)
based on the gross amount ofinterest thereto as defined
hereunder, which shall be paid by the borrower/issuer:
Provided, however, that in the case of a long-term
commercial paper whose maturity exceeds more than one
year, the borrower shall pay the tax based on the amount of
interest corresponding to one year, and thereafter shall pay
the tax upon accrual or actual payment (whichever is
earlier) of the untaxed portion of the interest which
corresponds to a period not exceeding one year.
The transaction tax imposed in this section shall be a
final tax to be paid by the borrower and shall be allowed as
a deductible item for purposes of computing the borrower's
taxable income.
Both the CTA and the Court of Appeals sustained the
assessment of transaction tax.
Picop reiterates its claim that it is exempt from the
payment of the transaction tax by virtue of its tax
exemption under R.A. No. 5186, as amended, known as the
Investment Incentives Act.
We agree with the CTA and the Court of Appeals that
Picop's tax exemption under R.A. No. 5186, as amended,
does not include exemption from the thirty-five percent
(35%) transaction tax. In the first place, the thirty-five
percent (35%) transaction tax is an income tax, that is, it is
a tax on the interest income of the lenders or creditors.
In Western Minolco Corporation v. Commissioner of
Internal Revenue it was stated that The 35% transaction
tax is imposed on interest income from commercial papers
issued in the primary money market. Being a tax on
interest, it is a tax on income.

Transaction tax is an income tax and as such,in any


event, falls outside the scope of the tax exemption granted
to registered pioneer enterprises by Section 8 of R.A. No.
5186, as amended. Picop was the withholding agent, obliged
to withhold thirty- five percent (35%) of the interest payable
to its lenders and to remit the amounts so withheld to the
Bureau of Internal Revenue ("BIR"). As a withholding agent,
Picop is made personally liable for the thirty- five percent
(35%) transaction tax and if it did not actually withhold
thirty-five percent (35%) of the interest monies it had paid
to its lenders, Picop had only itself to blame.
We conclude that Picop was properly held liable for
the thirty-five percent (35%) transaction tax due in respect
of interest payments onits money market borrowings.
WON PICOP is liable for interest and surcharge on
unpaid transaction tax NO
CIR prays that Picop be held liable for a twenty-five
percent (25%) surcharge and for interest at the rate of
fourteen percent (14%) per annum from the date prescribed
for its payment. In so praying, the CIR relies upon Section 10
of Revenue Regulation 7-77 which provided for these
penalities. CIR points to section 51 (e) of the 1977 Tax code
as its source of authority for assessing a surcharge and
penalty interest of 35% transaction tax due from Picop.
It will be seen that Section 51 (c) (1) and (e) (1) and
(3), of the 1977 Tax Code, authorize the imposition of
surcharge and interest only in respect of a "tax imposed by
this Title," that is to say, Title II on"Income Tax." It will also
be seen that Section 72 of the 1977 Tax Code imposes a
surcharge only in case of failure to file a return or list
"required by this Title," that is, Title II on "Income Tax." The
thirty-five percent (35%) transaction tax is, however,
imposed in the 1977 Tax Code by Section 210 (b) thereof
which Section is embraced in Title V on "Taxes on Business"
of that Code. Thus, while the thirty-five percent (35%)
transaction tax is in truth a tax imposed on interest income
earned by lenders or creditors purchasing commercial paper
on the money market, the relevant provisions, i.e., Section
210 (b), were not inserted in Title II of the 1977 Tax Code.
The end result is that the thirty-five percent (35%)

35

TAXATION 1 | B2015
CASE DIGESTS

transaction tax is not one of the taxes in respect of which


Section 51 (e) authorized the imposition of surcharge and
interest and Section 72 the imposition of a fraud surcharge.
It is not without reluctance that we reach the above
conclusion on the basis of what may well have been an
inadvertent error in legislative draftsmanship, a type of error
common enough during the period of Martial Law in our
country. Nevertheless, we are compelled to adopt this
conclusion. We consider that the authority to impose what
the present Tax Code calls (in Section 248) civil penalties
consisting of additions to the tax due, must be expressly
given in the enabling statute, in language too clear to be
mistaken. The grant of that authority is not lightly to be
assumed to have been made to administrative officials,
even to one as highly placed as the Secretary of Finance.
The state of the present law tends to reinforce our
conclusion that Section 51 (c) and (e) of the 1977 Tax Code
did not authorize the imposition of a surcharge and penalty
interest for failure to pay the thirty-five percent (35%)
transaction tax imposed under Section 210 (b) of the same
Code.
In other words, Section 247 (a) of the current NIRC
supplies what did not exist back in 1977 when Picop's
liability for the thirty-five percent (35%) transaction tax
became fixed. We do not believe we can fill that legislative
lacuna by judicial fiat. There is nothing to suggest that
Section 247 (a) of the present Tax Code, which was inserted
in 1985, was intended to be given retroactive application by
the legislative authority.
WON PICOP is liable for documentary and science
stamp tax YES
Picop
issued
sometime
in
1977
long-term
subordinated convertible debenture bonds with an
aggregate face value of P100,000,000.00. The proceeds of
the debenture bonds were in fact utilized to finance the BOIregistered operations of Picop. The CIR assessed
documentary and science stamp taxes, amounting to
P300,000.00, on the issuance of Picop's debenture bonds. It
is claimed by Picop that its tax exemption includes

exemption from the documentary and science stamp taxes


imposed under the NIRC.
CIR stresses that the tax exemption under the
Investment Incentives Act may be granted or recognized
only to the extent that the claimant Picop was engaged in
registered operations. The borrowing of funds from the
public, in the submission of the CIR, was not an activity
included in Picop's registered operations. CTA adopted the
view of the CIR.
Court of Appeals took a less rigid view of the ambit of
the tax exemption granted to registered pioneer enterprises.
It reasoned out that Since the money raised thereby was to
be used in its registered operation, PICOP should enjoy the
incentives granted to it by R.A. 5186, one of which is the
exemption from payment of all taxes under the National
Internal Revenue Code, except income taxes, otherwise the
purpose of the incentives would be defeated. Documentary
and science stamp taxes on debenture bonds are certainly
not income taxes.
SC agrees with CA. Tax exemptions are, to be sure, to
be "strictly construed," that is, they are not to be extended
beyond the ordinary and reasonable intendment of the
language actually used by the legislative authority in
granting the exemption. We consider that the actual
dedication of the proceeds of the bonds to the carrying out
of Picop's registered operations constituted a sufficient
nexus with such registered operations so as to exempt Picop
from stamp taxes ordinarily imposed upon or in connection
with issuance of such bonds.
Also, after commencement of the present litigation
before the CTA, the BIR took the position that the tax
exemption granted by R.A. No. 5186, as amended, does
include exemption from documentary stamp taxes on
transactions entered into by BOI-registered enterprises.
Similarly, in BIR Ruling No. 013, dated 6 February
1989, the Commissioner held that a registered pioneer
enterprise producing polyester filament yarn was entitled to
exemption "from the documentary stamp tax on [its] sale of
real property in Makati up to December 31, 1989."

36

TAXATION 1 | B2015
CASE DIGESTS

WON PICOP is entitled to deduct against current


income interest payments on loans for the purchase
of machinery and equipment YES
In 1969, 1972 and 1977, Picop obtained loans from
foreign creditors in order to finance the purchase of
machinery and equipment needed for its operations. In its
1977 Income Tax Return, Picop claimed interest payments
made in 1977, amounting to P42,840,131.00, on these loans
as a deduction from its 1977 gross income.
The CIR disallowed this deduction upon the ground
that, because the loans had been incurred for the purchase
of machinery and equipment, the interest payments on
those loans should have been capitalized instead and
claimed as a depreciation deduction taking into account the
adjusted basis of the machinery and equipment (original
acquisition cost plus interest charges) over the useful life of
such assets.
Both the CTA and the Court of Appeals sustained the
position of Picop and held that the interest deduction
claimed by Picop was proper and allowable.
Via Section 30 of the 1977 Tax Code, interest
payments on loans incurred by a taxpayer are allowed by
the NIRC as deductions against the taxpayers gross income.
Thus, the general rule is that interest expenses are
deductible against gross income and this certainly includes
interest paid under loans incurred in connection with the
carrying on of the business of the taxpayer. In the instant
case, the CIR does not dispute that the interest payments
were made by Picop on loans incurred in connection with
the carrying on of the registered operations of Picop. Neither
does the CIR deny that such interest payments were legally
due and demandable under the terms of such loans, and in
fact paid by Picop during the tax year 1977.
The CIR has been unable to point to any provision of
the 1977 Tax Code or any other Statute that requires the
disallowance of the interest payments made by Picop. The
CIR invokes Section 79 of Revenue Regulations No. 2. We
read the above provision of Revenue Regulations No. 2 as
referring to so called "theoretical interest". Such
"theoretical" or imputed interest does not arise from a

legally demandable interest-bearing obligation incurred by


the taxpayer who however wishes to find out. One thing that
Section 79 quoted above makes clear is that interest which
does constitute a charge arising under an interest-bearing
obligation is an allowable deduction from gross income.
CIR argues finally that to allow Picop to deduct its
interestpayments against its gross income would be to
encourage fraudulent claims to double deductions from
gross income.
Picop has not claimed to be entitled to double
deduction of its 1977 interest payments. The CIR has neither
alleged nor proved that Picop had previously adjusted its
cost basis for the machinery and equipment purchased with
the loan proceeds by capitalizing the interest payments here
involved.
We conclude that the CTA and the Court of Appeals
did not err in allowing the deductions of Picop's 1977
interest payments on its loans for capital equipment against
its gross income for 1977.
WON PICOP is entitled to deduct against current
income net operating losses incurred by Rustan Pulp
and Paper Mills NO
Picop entered into a merger agreement with the
Rustan Pulp and Paper Mills, Inc. ("RPPM") and Rustan
Manufacturing Corporation ("RMC"). Under this agreement,
the rights, properties, privileges, powers and franchises of
RPPM and RMC were to be transferred, assigned and
conveyed to Picop as the surviving corporation. RPPM and
RMC
were,
like
Picop,
BOI-registered
companies.
Immediately before merger effective date, RPPM had over
preceding years accumulated losses in the total amount of
P81,159,904.00. In its 1977 Income Tax Return, Picop
claimed P44,196,106.00 of RPPM's accumulated losses as a
deduction against Picop's 1977 gross income
Under our law, and outside the special realm of BOIregistered enterprises, there is no such thing as a carry-over
of net operating loss. To the contrary, losses must be
deducted against current income in the taxable year when
such losses were incurred. Moreover, such losses may be

37

TAXATION 1 | B2015
CASE DIGESTS

charged off only against income earned in the same taxable


year when the losses were incurred.
R.A. No. 5186 introduced the carry-over of net
operating losses as a very special incentive to be granted
only to registered pioneer enterprises and only with respect
to their registered operations. The statutory purpose here
may be seen to be the encouragement of the establishment
and continued operation of pioneer industries by allowing
the registered enterprise to accumulate its operating losses
which may be expected during the early years of the
enterprise and to permit the enterprise to offset such losses
against income earned by it in later years after successful
establishment and regular operations. To promote its
economicdevelopment goals, the Republic foregoes or
defers taxing the income of the pioneer enterprise until after
that enterprise has recovered or offset its earlier losses. We
consider that the statutory purpose can be served only if the
accumulated operating losses are carried over and charged
off against income subsequently earned and accumulated
by the same enterprise engaged in the same registered
operations.
In the instant case, to allow the deduction claimed by
Picop would be to permit one corporation or enterprise,
Picop, to benefit from the operating losses accumulated by
another corporation or enterprise, RPPM. RPPM far from
benefiting from the tax incentive granted by the BOI statute,
in fact gave up the struggle and went out of existence and
its former stockholders joined the much larger group of
Picop's stockholders.
WON PICOP IS ENTITLED TO DEDUCT AGAINST
CURRENT INCOME CERTAIN CLAIMED FINANCIAL
GUARANTEE EXPENSES NO
In its Income Tax Return for 1977, Picop also claimed a
deduction in the amount of P1,237,421.00 as financial
guarantee expenses. This deduction is said to relate to
chattel and real estate mortgages required from Picop by
the Philippine National Bank ("PNB") and DBP as guarantors
of loans incurred by Picop from foreign creditors.

CIR disallowed this claimed deduction upon the


ground of insufficiency of evidence. This disallowance was
sustained by the CTA and the Court of Appeals.
We consider that entitlement to Picop's claimed
deduction of P1,237,421.00 was not adequately shown and
that such deduction must be disallowed.
DISPOSITIVE: MODIFIED (see ruling part)

CIR V LEDNICKY
July 31, 1964
Reyes JBL J
Rods

SUMMARY: Spouses, resident aliens who derived income


solely within the Philippines, are claiming deductions from
their gross income the federal taxes that they paid to the
US. The SC held that these amounts are not deductible
from their income.
DOCTRINE:
The wording of section 30 (c) (1) (B) of the Internal
Revenue Act shows the laws intent that the right to deduct
income taxes paid to foreign government form the
taxpayers gross income is given only as an alternative or
substitute to his right to claim a tax credit for such foreign
income taxes under sec 30(c) (3) and (4), so that unless
the alien resident has a right to claim such tax credit if he
so chooses, he is precluded from deducting foreign income
taxes from his gross income.
FACTS:
Spouses Lednicky are American citizens residing in the
Philippines, and have derived all their income from
Philippine sources during the taxable years in question.
In 1956, they filed their income tax return for 1956, but later
on filed an amended tax return, which claimed deductions of
taxes paid to the US government as federal income

38

TAXATION 1 | B2015
CASE DIGESTS

tax for the same year. They requested a refund for such
alleged deductions. When the CIR failed to answer, they
filed a case with the Tax Court.
Another case was filed, wherein in 1955 they filed a tax
return, and also later amended it to a lesser net income.
However, CIR assessed a deficiency which was paid by
spouses. They then filed with the US Internal Revenue Agent
in Manila their federal income tax return on income from
Philippine sources on a cash basis. Again, this amounts were
added to their prayer for deductions.
Again in 1957, they filed returns, and later amended them
to include taxes paid to the US government on income
derived wholly from Philippine sources. They sought refund
once again
ISSUES: WON a citizen of the US residing in the Philippines,
may deduct from his gross income the income taxes he has
paid to the US government for the taxable year on the
strength of sec 30 (c-1)3 of the Philippine Internal
Revenue Code.
The Tax Court held that they may be deducted because
spouses did not signify in their income tax return a
desire to avail themselves of the benefits of par 3
3

SEC. 30. Deduction from gross income. In computing net income there
shall be allowed as deductions
(a) ...
(b) ...
(c) Taxes:
(1) In general. Taxes paid or accrued within the taxable year, except
(A) The income tax provided for under this Title;
(B) Income, war-profits, and excess profits taxes imposed by the authority
of any foreign country; but this deduction shall be allowed in the case of a
taxpayer who does not signify in his return his desire to have to any extent
the benefits of paragraph (3) of this subsection (relating to credit for foreign
countries);
(C) Estate, inheritance and gift taxes; and
(D) Taxes assessed against local benefits of a kind tending to increase the
value of the property assessed.

(B)4 of subsection. The tax credit is limited under par 4 (A


and B)5 of the same subsection.
RULING: No, he cannot.
RATIO:
The wording of section 30 (c) (1) (B) of the Internal Revenue
Act shows the laws intent that the right to deduct income
taxes paid to foreign government form the taxpayers gross
income is given only as an alternative or substitute to his
right to claim a tax credit for such foreign income taxes
under sec 30(c) (3) and (4), so that unless the alien resident
has a right to claim such tax credit if he so chooses, he is
precluded from deducting foreign income taxes from his
gross income. The wording that such deduction shall be
allowed in case of a taxpayer who does not signify his desire
to have the benefits of par 3, assumes that the taxpayer
may also signify his desire to claim a tax credit and waive
the deduction.
Had the law intended that foreign income taxes be deducted
from gross income in any event, regardless of the taxpayers
right to claim credit, it is the right to credit that should be
4

Par. (c) (3) Credits against tax for taxes of foreign countries. If the
taxpayer signifies in his return his desire to have the benefits of this
paragraph, the tax imposed by this Title shall be credited with
(A) ...;
(B) Alien resident of the Philippines. In the case of an alien resident of
the Philippines, the amount of any such taxes paid or accrued during the
taxable year to any foreign country, if the foreign country of which such
alien resident is a citizen or subject, in imposing such taxes, allows a similar
credit to citizens of the Philippines residing in such country;
5
Par. (c) (4) Limitation on credit. The amount of the credit taken under
this section shall be subject to each of the following limitations:
(A) The amount of the credit in respect to the tax paid or accrued to any
country shall not exceed the same proportion of the tax against which such
credit is taken, which the taxpayer's net income from sources within such
country taxable under this Title bears to his entire net income for the same
taxable year; and
(B) The total amount of the credit shall not exceed the same proportion of
the tax against which such credit is taken, which the taxpayer's net income
from sources without the Philippines taxable under this Title bears to his
entire net income for the same taxable year.

39

TAXATION 1 | B2015
CASE DIGESTS

conditioned upon the waiving of the deduction, and the right


to reduction under subsection (c-1-B) would have been
made absolute. The purpose of the law is to prevent the
taxpayer from claiming twice the benefits of his payment of
foreign taxes, by deduction from gross income and by tax
credit.
On the argument that the spouses would be subject to
double taxation, the SC said that double taxation is
obnoxious only when the taxpayer is taxed twice for the
benefit of the same governmental entity, but in this case the
Philippine government would only receive the proceeds of
one tax. The court said that between the US, where income
was not earned and the taxpayer does not reside, and the
Philippines, where the income was earned and the taxpayer
resides, justice and equity demand that the tax on the
income should accrue to the benefit of the Philippines. Any
relief from double taxation should come from the US, since
the US did not contribute to the production of wealth that is
being taxed.
The court also said that the interpretation of the spouses
would also place a resident alien with only domestic sources
of income in an equal or better position than one who has
both domestic and foreign sources of income, a situation
which is unfair and short of logic. Also, to allow such
interpretation would confer on the other government the
power to reduce the income tax to the Philippines by
increasing the tax rates on the alien resident. It would be
tantamount to subordinating our taxes to a foreign
government, which is incompatible with the status of the
Philippines as an independent and sovereign state.

FERNANDEZ HERMANOS V.
CIR
SEPTEMBER 30, 1969
Teehankee. J.

SUMMARY: Fernandez Hermanos Inc. was assessed by the


CIR with deficiency. Fernandez claimed for several
deductions, one of which was the losses incurred as
regards the money it gave to Palawan Manganese. The CIR
disallowed the deductions. The SC discussed the disputed
items in seriatim.
DOCTRINE: (topic: losses) It has been held that the
voluntary advances made without expectation of
repayment do not result in deductible losses.
FACTS: Fernandez Hermanos, Inc., is a domestic
corporation organized for the principal purpose of engaging
in business as an "investment company" with main office at
Manila. The Commissioner of Internal Revenue assessed
against the taxpayer as alleged deficiency income taxes for
the years 1950-1954. Fernandez claimed several
deductions, one of which was the losses or bad debt as
regards the funds it gave to Palawan Manganese Mines,
which were disallowed by the CIR.
The Tax Court sustained the Commissioner's disallowances
while overruled the others. Hence, both Fernandez and the
Commissioner appealed the decision.

DISPOSITIVE: Decisions of CTA reversed, disallowance of


refunds affirmed.

ISSUES: WON the disallowances should be allowed

GUTIERREZ V. COLLECTOR (RAFANAN)

RATIO:
Court discussed the disallowances in seriatim
1. Re allowances/disallowances of losses.

40

TAXATION 1 | B2015
CASE DIGESTS
a.

b.

Allowance of losses in Mati Lumber Co.


(1950). - There was adequate basis for the
writing off of the stock as worthless securities.
Assuming that the Company would later
somehow realize some proceeds from its
sawmill and equipment, which were still
existing as claimed by the Commissioner, and
that such proceeds would later be distributed
to its stockholders such as the taxpayer, the
amount so received by the taxpayer would
then properly be reportable as income of the
taxpayer in the year it is received.
(RELEVANT) Disallowance of losses in or bad
debts of Palawan Manganese Mines,
Inc. (1951) Palawan Manganese Mines, Inc.,
the controlling stockholders of which are also
the controlling stockholders of petitioner
corporation, requested financial help from
petitioner which the latter granted. The two
entered into and MOA.
i. In giving advances to Palawan
Manganese Mine Inc., petitioner did
not expect to be repaid.
1. The memorandum agreement
signed by the parties appears
to be very clear that the
consideration for the advances
made by petitioner was 15% of
the net profits of Palawan
Manganese Mines, Inc. In other
words, if there were no earnings
or profits, there was no
obligation to repay those
advances.
2. It has been held that the
voluntary advances made
without expectation of
repayment do not result in
deductible losses.

3. No bad debt could arise where


there is no valid and subsisting
debt.
ii. The funds given were not loans but
investments
1. This fact explains the liberality
with which the taxpayer made
such large advances to the
subsidiary, despite the latter's
admittedly poor financial
condition.
iii. The Tax Court correctly held that the
subsidiary company was still in
operation in 1951 and 1952 and the
taxpayer continued to give it advances
in those years, and, therefore, the
alleged debt or investment could not
properly be considered worthless and
deductible in 1951, as claimed by the
taxpayer. Furthermore, neither under
Section 30 (d) (2) of our Tax Code
providing for deduction by
corporations of losses actually
sustained and charged off during the
taxable year nor under Section 30 (e)
(1) thereof providing for deduction of
bad debts actually ascertained to be
worthless and charged off within the
taxable year, can there be a partial
writing off of a loss or bad debt, as was
sought to be done here by the
taxpayer. For such losses or bad debts
must be ascertained to be so and
written off during the taxable year, are
therefore deductible in full or not at all,
in the absence of any express

41

TAXATION 1 | B2015
CASE DIGESTS

provision in the Tax Code authorizing


partial deductions.
c.
Disallowance of losses in Balamban Coal
Mines (1950 and 1951) - the losses "are
deductible in 1952, when the mines were
abandoned, and not in 1950 and 1951, when
they were still in operation."
d.
and e. Allowance of losses in Hacienda
Dalupiri (1950 to 1954) and Hacienda Samal
(1951-1952). - Hacienda Dalupiri was
operated by petitioner for business and not
pleasure. It was mainly a cattle farm, although
a few race horses were also raised. It does not
appear that the farm was used by petitioner
for entertainment, social activities, or other
non-business purposes. Therefore, it is
entitled to deduct expenses and losses in
connection with the operation of said farm.
2. Disallowance of excessive depreciation of buildings
(1950-1954). - the taxpayer did not submit adequate
proof of the correctness of the taxpayer's claim that
the depreciable assets or buildings in question had a
useful life only of 10 years so as to justify its 10%
depreciation per annum claim, such finding being
supported by the record.
3. Taxable increase in net worth (1950-1951)
a.
The principle underlying the taxability of an
increase in the net worth of a taxpayer rests
on the theory that such an increase in net
worth, if unreported and not explained by the
taxpayer, comes from income derived from a
taxable source.
b.
In this case, the increase in the net worth of
petitioner for 1950 to the extent of
P30,050.00 was not the result of the receipt

by it of taxable income. It was merely the


outcome of the correction of an error in the
entry in its books relating to its indebtedness
to the Manila Insurance Company.
4. Gain realized from sale of real property (1950) The
discrepancy in Fernandez reported tax as regards
the properties acquired was properly accounted for
by the improvements made on the property.
5. Allowance of losses in Hacienda Dalupiri (1957) the
contention on this item revolves mainly on the
accounting method employed by Fernandez. The
hybrid method which was used was justified because
the company was engaged in more than one trade.
6. Disallowance of amortization of alleged "contractual
rights."
a.
Palawan ceded its assets to Fernandez as
partial settlement of its indebtedness
b.
Fernandez determined the cost of the mines
by adding the value of the contractual rights
and the value of its mining claims. CIR
disallowed the deduction.
c.

The law in point is Section 30(g) (1) (B) of the


Revenue Code, before its amendment by
Republic Act No. 2698, which provided in part:

"(g) Depletion of oil and gas wells and mines.:


"(1) In general. ... (B) in the case of mines, a
reasonable allowance for depletion thereof not to
exceed the market value in the mine of the product
thereof, which has been mined and sold during the
year for which the return and computation are made.
The allowances shall be made under rules and
regulations to be prescribed by the Secretary of
Finance: Provided, That when the allowances shall
equal the capital invested, ... no further allowance
shall be made."

42

TAXATION 1 | B2015
CASE DIGESTS

exhaust his remedies first to recover or reduce his loss.


DISPOSITIVE: ACCORDINGLY, the judgment of the Court of
Tax Appeals, subject of the appeals in Cases Nos. L-21551
and L-21557, as modified by the crediting of the losses of
P36,722.42 disallowed in 1951 and 1952 to the taxpayer for
the year 1953 as directed in paragraph 1 (c) of this decision,
is hereby affirmed. The judgment of the Court of Tax Appeals
appealed from in Cases Nos. L-24972 and L-24978 is
affirmed in toto. No costs. So ordered.

PLARIDEL vs. CIR


Dec 11, 1967
Justice Bengzon
tecskat

SUMMARY:
Plaridel Surety executed a bond in favor of Constancio
San Jose to secure his obligation to PL Galang Machinery.
As additional security, Plaridel also required San Jose and
one Cuervo to execute an indemnity agreement in favor or
Plaridel Surety. San Jose failed to perform his obligation. In
the suit filed by PL Galang, Plaridel was adjudged solidarily
liable with San Jose in favor of PL Galang. For that year,
Plaridel deducted the amount it paid from its gross income.
The CIR adjudged it an income deficiency tax. The Court
held that the amount paid to Pl Galang was not a
deductible loss since the amount was recoverable from San
Jose and Cuervo.
DOCTRINE: Deductible Loss
The rule is that loss deduction will be denied if there is a
measurable right to compensation for the loss, with
ultimate collection reasonably clear. So where there is
reasonable ground for reimbursement, the taxpayer must
seek his redress and may not secure a loss deduction until
he establishes that no recovery may be had. In other
words, as the Tax Court put it, the taxpayer (Plaridel) must

FACTS:
1. Petitioner Plaridel Surety & Insurance Co is a domestic
corp engaged in the bonding business. Plaridel, as
surety, and Constancio San Jose, as principal, executed
a bond in favor of PL Galang Machinery Co to secure
San Joses contractual obligation to produce and supply
logs to PL Galang.
2. As additional security, Plaridel required San Jose and
one Ramon Cuervo to execute an indemnity
agreement. Accordingly, San Jose constituted a chattel
mortgage on its logging machineries and other
movables in Plaridel's favor while Ramon Cuervo
executed a real estate mortgage.
3. San Jose was not able to deliver the logs to PL Galang.
In 1953, the CFI adjudged San Jose and Palridel liable.
It also directed Cuervo to reimburse Plaridel for
whatever amount it would pay PL Galang. The CA
affirmed the CFIs decision. Plaridel paid P44,490 to PL
Galang in 1957.
4. For its income tax return for the year 1957, Plaridel
claimed the amount of P44k from its gross income.
5. The CIR disallowed the claimed deduction and
assessed Plaridel an income deficiency tax.
ISSUE:
1. WON the amount paid to PL Galang is a deductible
loss.-NO
RATIO:
1. Breakdown: Of the sum of P44,490.00, the amount of
P30,600.00 which is the principal sum stipulated in
the performance bond is being claimed as loss
deduction under Sec. 30 (d) (2) of the Tax Code and
P10,000.00 which is the interest that had accrued
on the principal sum is now being claimed as
interest deduction under Sec. 30 (b) (1).
2. Loss is deductible only in the taxable year it actually
happens or is sustained. However, if it is

43

TAXATION 1 | B2015
CASE DIGESTS

3.

4.

5.

6.

compensable by insurance or otherwise, deduction


for the loss suffered is postponed to a subsequent
year, which, to be precise, is that year in which it
appears that no compensation at all can be had, or
that there is a remaining or net loss, i.e., no full
compensation.
Plaridel effected payment to Galang Machinery
pursuant to a final decision in 1957. However, under
the same court decision, San Jose and Cuervo were
obligated to reimburse Plaridel for whatever
payments it would make to Galang Machinery.
Clearly, Plaridel's loss is compensable otherwise
(than by insurance). It should follow, then, that the
loss deduction can not be claimed in 1957.
The rule is that loss deduction will be denied if
there is a measurable right to compensation
for
the
loss,
with
ultimate
collection
reasonably clear. So where there is reasonable
ground for reimbursement, the taxpayer must
seek his redress and may not secure a loss
deduction until he establishes that no recovery
may be had. In other words, as the Tax Court
put it, the taxpayer (Plaridel) must exhaust his
remedies first to recover or reduce his loss.
In the case at bar, Plaridel did not exhaust its
remedies. However, even assuming that there was
no reasonable expectation of recovery, still no loss
deduction can be had. Sec. 30 (d) (2) of the Tax Code
requires a charge-off as one of the conditions for loss
deduction:
In the case of a corporation, all losses actually
sustained and charged-off within the taxable year
and not compensated for by insurance or otherwise.
A requisite of a charge-off is found under Section
23(f) of the US Internal Revenue Code of 1939 which
states: In the case of a corporation, losses sustained
during the taxable year and not compensated for by
insurance or otherwise. Plaridel, who had the
burden of proof failed to adduce evidence that there

was a charge-off in connection with the P44,490.00


or P30,600.00 which it paid to Galang Machinery.
7. In connection with the claimed interest deduction of
P10,000.00, the Solicitor General correctly points out
that this question was never raised before the Tax
Court The alleged interest deduction not having been
properly litigated as an issue before the Tax Court, it
is now too late to raise and assert it before this
Court.
RULING: The amount paid to Pl Galang is not a deductible
loss.
DISPOSITIVE: Appealed decision is affirmed.

CHINA BANKING VS. CA


July 19, 2000
J. Vitug
Mabel Africa

SUMMARY: Petitioner wished to exclude from its tax


return the investment it made to the First CBC Capital who
has become bankrupt. China Banking argued that the
investment became bad debt. The Commissioner, CTA and
the SC disallowed such exclusion on the ground that is not
an ordinary loss and not subject to repayment.
DOCTRINE:
1. The equity investment in shares of stock held by
the China Banking is not an indebtedness, and it is
a capital, not an ordinary, asset.
2. Assuming that the investment indeed became
worthless, the loss sustained is a capital, not an
ordinary, loss.
3. The capital loss sustained by China Banking can
only be deducted from capital gains if any derived
by it during the same taxable year that the
securities have become worthless.

44

TAXATION 1 | B2015
CASE DIGESTS

FACTS:
1. China Banking Corporation made a 53% equity
investment in the First CBC Capital (Asia) Ltd.
a. Investment made in the year 1980
b. First CBC Capital is a Hongkong subsidiary
engaged in financing and investment with
deposit-taking function
c. China Bankings investment amounted to Php
16, 227, 851.80 (consisting of 106,000 shares
with a par value of Php 100/share)
2. A regular examination of the financial books &
investment portfolios of China Banking was
conducted by Bankgo Sentral in 1986
a. It was shown that First CBC Capital has
become insolvent
b. China Banking then wrote-off as worthless its
investment in First CBC Capital in its 1987 ITR
i. With the approval of Bangko Sentral
ii. Treated it as a bad debt or as an
ordinary loss deductible from its gross
income
3. The Commissioner disallowed the deduction
a. Assessed China Banking for tax deficiency of
Php 8,533,328.04
b. Grounds for disallowing the deduction
i. The
investment
should
not
be
classified as being worthless
ii. Even if the license of First CBC Capital
as a deposit-taping has been
revoked, it can still exercise its
financing and investment activities
iii. Even if we assume that the securities
became worthless, they should be
classified as capital loss instead of a
bad
1. This is due to the reason that
there is no indebtedness to
speak
of
between
China
Banking & its subsidiary
4. The CTA sustained the decision of the Commissioner

5. Claim of China Banking


a. Bases that the shares of stock have become
worthless
i. Profit and Loss Account for the YearEnd (Dec. 31, 1987)
ii. Recommendation of Bangko Sentral
that the equity investment be writtenoff due to the insolvency of the
subsidiary
ISSUES: WON the shares of stock in question may be
deducted?
RATIO: No.
RULING:
1. Equity investment
a. Definition: is a capital, not ordinary, asset of
the investor the sale or exchange of which
results in either a capital gain or a capital loss
b. The gain or loss is ordinary when the property
sold or exchanged is not a capital asset
c. Capital asset defined in Sec. 33 (1) of NIRC
2. Shares of stock
a. From the above premises, it can be concluded
that shares of stock, like other securities
stated in Sec. 20 (t) of NIRC, would be
ordinary asset only to a dealer in securities or
a person engaged in the purchase and sale of,
or an active trader (for his own account).
b. If the shares of stock are held by way of an
investment, the shares to him would be
capital assets.
3. On loss sustained by the holder of the securities
a. If securities are capital assets to him, loss is
treated as a capital loss as if incurred from a
sale or exchange transaction
b. 2 conditions that must concur to result a
capital gain/capital loss
i. There is a sale or exchange

45

TAXATION 1 | B2015
CASE DIGESTS

ii. The thing sold or exchanged is a


capital asset
c. When securities become worthless no sale
or exchange loss from the sale or exchange
of capital assets
d. Capital losses are allowed to be deducted only
to the extent of capital gains
4. Case at bar
a. First CBC Capital (investee) is a subsidiary
corporation of China Banking
b. China Bankings shares in the investee are not
intended for purchase/sale BUT as an
investment
i. Effect: loss is a capital loss
c. The shares of stock involved is not a bad debt
i. They are not subject to obligatory
repayment an element of a bad debt
ii. Basis: Sec. 33, NIRC
DISPOSITIVE: The petition is denied. The decision of the CA
disallowing the claimed deduction of Php 16, 227, 851.80 is
affirmed.

COLLECTOR v. GOODRICH
22 December 1967

representation expenses. Respondent filed a Motion for


Reconsideration and New Trial. The CTA, on 19 November
1963, amended its decision and allowed the deduction for
representation expenses. The CIR appealed to the SC. The
SC held that only 8 of the alleged bad debts were deductible
since they were the only ones shown to be worthless; the
others were not allowed. As for the alleged representation
expenses, the Court held that the Respondents failed to
produce receipts proving that the expenses were actually
incurred; hence, these deductions must not be allowed.
DOCTRINE:
When an event subsequently occurs that is inconsistent with
an earlier deduction, which appears to be properly applied,
the taxpayer is liable for deficiency in taxes.
As regards bad debts:
To be entitled to deduction of bad debts, two facts must be
proven: 1) that the taxpayer did in fact ascertain the debt to
be worthless in the year for which deduction is sought; and
2) that in so doing, he acted in good faith. The Court further
explained that good faith is not enough. The claimant must
also show that he had reasonably investigated the relevant
facts and had drawn a reasonable inference from the
information thus obtained by him.
As regards representation expenses:
The claimant must be able to show receipts which prove that
the expenses were actually incurred in payment to the
entities that issued the same.

CJ Concepcion
Dan Amorin

SUMMARY:
CIR assessed Goodrich for deficiency income taxes for 1951
and 1952. These were based on the disallowed deductions
claimed by Respondent for several alleged bad debts for
1951, and alleged representation expenses in 1952.
Goodrich appealed the assessments to the CTA, which
rendered a decision on 8 June 1963 allowing the deduction
of bad debts but disallowing the deduction of alleged

FACTS:
Petitioner CIR assessed Respondent Goodrich International
Rubber Company for deficiency income taxes for the years
1951 and 1952 in the sums 14 128 and 8 439,
respectively. These assessments were based on disallowed
deductions claimed by Goodrich: several alleged bad debts
for 1951 (total of 50 455.41) and representation expenses
allegedly incurred in 1952 (total of 30 138.88).

46

TAXATION 1 | B2015
CASE DIGESTS

The breakdown of bad debts are:


1. Portillos Auto Seat Cover
2. Visayan Rapid Transit
810.26
3. Bataan Auto Seat Cover
373.31
4. Tres Amigos Auto Supply
330.31
5. P.C. Teodoro
6. Ordinance Service, P.A.
386.42
7. Ordinance Service, P.C.
796.26
8. National Land Settlement Administration
020.76
9. National Coconut Corporation
644.74
10. Interior Caltex Service Station
505.87
11. San Juan Auto Supply
530.64
12. PACSA
13. Philippine Naval Patrol
14.18
14. Surplus Property Commission
277.68
15. Alverez Auto Supply
16. Lion Shoe Store
686.93
17. Ruiz Highway Transit
18. Esquire Auto Seat Cover
536.94
TOTAL:
455.41

630.31
17

1
650

1. Expenses at Elks Club


10 959.21
2. Manila Polo Club
947.35
3. Army and Navy Club

812.95
4. Manila Golf Club
478.45
5. Wack Wack Golf Club, Casino Espaol, etc.
940.92
TOTAL:
138.88

45.36

285.62
1
2 350
3

The breakdown of alleged representation expenses are:

50

4
2
4
6

30

Goodrich appealed the assessments to CTA. On 8 June 1963,


the appellate court rendered a decision allowing the
deduction of bad debts, but disallowing the alleged
representation expenses. Goodrich filed a Motion for
Reconsideration and New Trial and the CTA, on 19 November
1963, amended its decision and allowed the deductions for
representation expenses. The CIR then appealed the
decision to the SC.
ISSUES:
1. Whether or not the debts had been properly deducted as
bad debts for the year 1951
2. Whether or not Respondent is entitled to its claims of
deduction for alleged representation expenses
RATIO:
1. Only those which Goodrich was able to show as
worthless should be allowed.
2. No. The CTA should have not allowed deduction.
RULING:

47

TAXATION 1 | B2015
CASE DIGESTS

1. As for the bad debts, the Court examined each item in


the list. The first ten were rejected:
A. Portillos Auto Seat Cover debt incurred in 1950 and
was written off as bad debt that same year after
debtor paid 70; debtor paid full balance in 1952
B. Visayan Rapid Transit debt incurred in 1950 and
was charged off as bad debt in 1951 after debtor
paid 275.21; no other payment was made since
debtor became insolvent, but no proof was
introduced
C. Bataan Auto Seat Cover balance of a debt of
474.13 contracted in 1949; debtor paid 100 in
1951 and the balance was charged off as bad debt
that same year; in 1952, debtor paid an additional
50
D. Tres Amigos Auto Supply outstanding account since
1949 and has not been collected
E. P.C. Teodoro - 751.91 in 1949; debtor paid 101.91
in 1951, which left a balance of 650 which was
charged off as bad debt in that same year; in 1952,
debtor paid 150
F. Ordinance Service, P.A. outstanding account of
government agency in 1949 was 817.55; in 1951,
debtor paid 431.13 and the balance of 386.42
was written off as bad debt that same year
G. Ordinance Service, P.C. in 1950, account was
796.26; account was written off as bad debt in 1951;
in 1952, debtor paid in full
H. National
Land
Settlement
Administration

outstanding account in 1949 was 7 041.51; in


1951, debtor paid 4 020.75 leaving a balance of
3 020.76 that was written off as bad debt that same
year; office was under liquidation, and the Board of
Liquidators promised to pay when funds become
available

I.

J.

National Coconut Corporation outstanding account


since 1949; written off as bad debt in 1951; while
under liquidation, its Board of Liquidators promised
to pay upon availability of funds; in 1961, debt was
fully paid
Interior Caltex Service Station account was 2
705.87 in 1950; debtor made partial payments so
that as of December 1951, balance was 1505.87;
debtor pad 200 in 1952, 113.20 in 1954, 750 in
1961, and 300 in 1962; account had been written
off as bad debt in 1951

The Court held that Goodrich was not able to establish


that these debts were actually worthless or that it had
reasonable grounds to be so in 1951. What the law
provides is the deduction of debts actually ascertained
to be worthless within the taxable year. This is to
prevent arbitrary action by taxpayer and unduly avoid
tax liability.
The Court held that to be entitled to deduction of bad
debts, two facts must be proven: 1) that the taxpayer
did in fact ascertain the debt to be worthless in the year
for which deduction is sought; and 2) that in so doing, he
acted in good faith. The Court further explained that
good faith is not enough. The claimant must also show
that he had reasonably investigated the relevant facts
and had drawn a reasonable inference from the
information thus obtained by him. In the instant case,
Respondent has failed to show this; Goodrich has not
shown evidence that the debtors could not pay the debts
that they were worthless.
The Court also found the fact that payments were made
after some of the accounts had been characterized as
bad debts shows the undue haste with which they had

48

TAXATION 1 | B2015
CASE DIGESTS

been written off. The Court also notes that Respondent


failed to attach to its income tax returns a statement
showing the propriety of the deductions therein made for
the alleged bad debts, making its omission a violation of
Revenue Regulations No. 2, Section 102.

were in strained financial condition and had no


attachable or leviable property (Lion was burned
twice in 1948 and 1949 and later went out of
business after operating on limited scale after the
incidents; Ruiz had more debts than assets)

On the other hand the last eight were, according to the


Court, properly written off:

The deduction of these eight accounts, aggregating 22


627.35, as bad debts should be allowed.

K. San Juan Auto Supply account contracted in 1950;


Respondents counsel failed to secure payment in
November 1950, a suit for collection was filed but
debtor did not appear at the hearing of the case;
judgment rendered in favour of the creditor in 1951
but the corresponding writ of execution was returned
unsatisfied for no properties could be attached or
levied upon.
L. PACSA (together with M. Philippine Naval Patrol, N.
Surplus Property Commission, and O. Alvarez Auto
Supply) these accounts were 2 or 3 years old in
1951; Respondent failed to collect the amounts
despite letters of demand; considering the small
amounts involved, Goodrich was justified in feeling
that the unsuccessful efforts therefore exerted to
collect the same sufficed to warrant their being
written off
P. Lion Shoe Store (together with Q. Ruiz Highway
Transit, and R. Esquire Auto Seat Cover) up to 1951,
when they were written off, counsel had sent 17
letters of demand to Lion, 16 to Ruiz, and 6 to
Esquire; in 1951 debtors made the following partial
payments: 1 050 (Lion), 400 (Ruiz), and 300
(Esquire); subsequent to write-off, additional small
payment were made; counsel for Respondent
investigated their ability to pay and found that they

2. The claim for the deduction of alleged representation


expenses is based upon the receipts issued, not by the
entities in which the alleged expenses had been
incurred, but by the officers of Goodrich who allegedly
paid them. The Court held that this claim must be
rejected. If these expenses had really been incurred,
Goodrich would have been able to produce the receipts
issued by the entities to which the payments had been
made. These issued by officers merely attest to their
claim that they had incurred and paid said expenses;
they do not establish payment of expenses to the
entities in which the same are said to have been
incurred. The CTA erred in allowing the deduction
thereof.
DISPOSITIVE:
Petition partly granted. The alleged representation expenses
totally disallowed. The claim for bad debts allowed up to the
sum of 22 627.35 only. No pronouncements as to costs.

BASILAN ESTATES INC V.


CIR
September 5, 1967
Bengzon, J.

49

TAXATION 1 | B2015
CASE DIGESTS
S. J. Lora (note: I kept the doctrinal part re: depreciation in
verbatim)

SUMMARY: On Mar. 24, 1954, Basilan Estates Inc. filed its


income tax returns for 1953 and paid its income tax. Based
on examiners report, the CIR assessed Basilan Estates a
deficiency income tax for 1953 as well as 25% surtax on
unreasonably accumulated profits. The latter did not pay
the said amount, so a warrant of distraint and levy was
issued. Basilan Estates then filed a petition for review
before the CTA, alleging that the period of assessment and
collection has already prescribed, that there was error in
disallowing
claimed
depreciations,
traveling
and
miscellaneous expenses, as well as in finding the that there
was unreasonably accumulated profits and the imposition
of 25% surtax thereon. The CTA found that there was no
prescription and affirmed the deficiency assessment. Thus,
Basilan appealed to the SC. The SC modified the judgment
appealed from to the extent that petitioner is allowed its
deductions for travelling and miscellaneous expenses, but
affirmed insofar as the petitioner is liable for deficiency
income tax for 1953 and 25% surtax on the unreasonably
accumulated profit.
DOCTRINE: Depreciation is the gradual diminution in the
useful value of tangible property resulting from wear and
tear and normal obsolescense. The term is also applied to
amortization of the value of intangible assets, the use of
which in the trade or business is definitely limited in
duration. Depreciation commences with the acquisition of
the property and its owner is not bound to see his property
gradually waste, without making provision out of earnings
for its replacement. It is entitled to see that from earnings
the value of the property invested is kept unimpaired, so
that at the end of any given term of years, the original
investment remains as it was in the beginning. It is not
only the right of a company to make such a provision, but
it is its duty to its bond and stockholders, and, in the case
of a public service corporation, at least, its plain duty to
the public.

The income tax law does not authorize the depreciation of


an asset beyond its acquisition cost. Hence, a deduction
over and above such cost cannot be claimed and allowed.
The reason is that deductions from gross income are
privileges, not matters of right. They are not created by
implication but upon clear expression in the law.
FACTS:
On Mar. 24, 1954, Basilan Estates Inc. filed its income
tax returns for 1953 and paid an income tax of P8028.
Based on examiners report, the CIR assessed Basilan
Estates a deficiency income tax of P3912 for 1953 and
P86,876.85 as 25% surtax on unreasonably accumulated
profits. The latter did not pay the said amount, so a warrant
of distraint and levy was issued. However, this was not
executed because Basilan Estates was able to convince the
Deputy Commissioner of Internal Revenue to order the
Director of the district in Zamboanga City to just maintain
constructive embargo.
Basilan Estate requested for reinvestigation but this did
not push through due to its refusal to waive the period of
prescription. Thus, it was served a notice that the warrant of
distraint and levy would be executed. Basilan Estates then
filed a petition for review before the CTA, alleging that the
period of assessment and collection has already prescribed,
that there was error in disallowing claimed depreciations,
traveling and miscellaneous expenses, as well as in finding
the that there was unreasonably accumulated profits and
the imposition of 25% surtax thereon. The CTA found that
there was no prescription and affirmed the deficiency
assessment. Thus, Basilan appealed to the SC.
ISSUES:
1. Whether the Commissioner has the right to collect
deficiency income tax that has prescribed
2. Whether depreciation shall be determined on the
acquisition cost or on the reappraised value of the assets
3. Whether the disallowance of items claimed as deductible
was proper

50

TAXATION 1 | B2015
CASE DIGESTS

4. Whether there was unreasonably accumulated profits


5. Whether the petitioner is exempt from the penalty tax
under RA 1823 amending Sec. 25 of the Tax Code
RATIO:
1. The Commissioner collected the deficiency before the
time of prescription.
2. The income tax law does not authorize the depreciation
of an asset beyond its acquisition cost
3. NO. Taxpayers stand on the issue is sustained.
4. YES. There was unreasonable accumulation.
5. NO. The unreasonable accumulation was in 1953; the
exemption mentioned above was amended more than 3
years after the period covered by the assessment.
RULING:
1. Petitioner: Claimed that it did not receive notice of the
assessment of the deficiency tax, or if it did, the receipt
was beyond the 5-year prescriptive period. Petitioners
defense is an annotation on the notice no
accompanying letter 11/25, which is supposed to
indicate that they received the notice after Mar. 24 1959,
the last date of the 5-year period within which deficiency
tax could be assessed (because the original tax returns
were filed 5 years earlier).
SC: Does not accept petitioners interpretation,
considering the presumption of regularity in the
performance of official functions. The notice of
assessment showed that it was made on Feb. 26, 1959,
which is within the 5-year prescription period. Moreover,
there were other evidences which establish that notice
of assessment was already sent: letters from the
Commissioner and the Regional Director saying that the
notice of assessment was already sent, as well as an
indorsement of the case of the Chief of the Law Division
where it was alleged that notice was already sent.
Even if the notice was received late by the petitioner,
under Sec. 331 of the Tax Code which requires 5 years

within which to assess deficiency taxes, the assessment


is deemed made when notice to this effect is released,
mailed or sent by the Collector to the taxpayer and it is
not required that the notice be received by the taxpayer
within the aforementioned five-year period.
2. Petitioner: Claimed deductions for the depreciation of
its assets up to 1949 based on their acquisition costs. It
increased the depreciable value of the assets as of Jan.
1, 1950 to conform w/ the increase in cost for their
replacement. Thus, from 1950-1953, it made deductions
from the gross income the value of depreciation that was
computed on the reappraised value.
CIR: Found that the reappraised assets depreciated in
1953 were the same as those claimed in 1952. For 1952,
the depreciation allowable on said assets were
determined to be P36,842.04 computed on their
acquisition cost at rates fixed by the taxpayer. The
Commissioner pegged the deductible depreciation for
1953 on the same old assets at P36,842.04 and
disallowed the excess thereof in the amount of
P10,500.49.
SC: The question for resolution is whether depreciation
shall be determined on the acquisition cost or on the
reappraised value of the assets.
Depreciation is the gradual diminution in the useful
value of tangible property resulting from wear and tear
and normal obsolescense. The term is also applied to
amortization of the value of intangible assets, the use of
which in the trade or business is definitely limited in
duration. Depreciation commences with the acquisition
of the property and its owner is not bound to see his
property gradually waste, without making provision out
of earnings for its replacement. It is entitled to see that
from earnings the value of the property invested is kept
unimpaired, so that at the end of any given term of
years, the original investment remains as it was in the

51

TAXATION 1 | B2015
CASE DIGESTS

beginning. It is not only the right of a company to make


such a provision, but it is its duty to its bond and
stockholders, and, in the case of a public service
corporation, at least, its plain duty to the public.
Accordingly, the law permits the taxpayer to recover
gradually his capital investment in wasting assets free
from income tax. Precisely, Section 30 (f) (1) allows a
deduction from gross income for depreciation but limits
the recovery to the capital invested in the asset being
depreciated.
The income tax law does not authorize the depreciation
of an asset beyond its acquisition cost. Hence, a
deduction over and above such cost cannot be claimed
and allowed. The reason is that deductions from gross
income are privileges, not matters of right. They are not
created by implication but upon clear expression in the
law.
Moreover, the recovery, free of income tax, of an
amount more than the invested capital in an asset will
transgress the underlying purpose of a depreciation
allowance. For then what the taxpayer would recover will
be, not only the acquisition cost, but also some profit.
Recovery in due time thru depreciation of investment
made is the philosophy behind depreciation allowance;
the idea of profit on the investment made has never
been the underlying reason for the allowance of a
deduction for depreciation.
The claim for depreciation beyond P36,842.04 or in the
amount of P10,500.49 has no justification in the law. The
determination, therefore, of the CIR disallowing said
amount, affirmed by the CTA, is sustained.
3. Several of petitioners expenses were disallowed
because they could not be explained nor were they
supported by documents.
Petitioner: The petitioners accountant explained that
these expenses were trip and travel expenses of the

president, but did not have receipts because these were


burned during the Basilan fire. These were also not
included when it sent its records to Manila because 5
years had already lapsed when these expenses were
incurred.
SC: Sustained petitioners arguments because under
Sec. 337 of the Tax Code, receipts and papers supporting
such expenses need only be kept by the taxpayer for a
period of 5 years from the last entry.
4. CIR: Found that petitioner had violated of Sec. 25 of the
Tax Code when it unreasonably accumulated profits as of
1953 in the amount of P347,507.01, based on certain
circumstances.
Petitioners arguments:
1. Tried to show that when the surplus was being
considered, the examiner failed to account the
possible expenses for cultivation, labor, fertilitation,
drainage, irrigation, repair, etc.
2. Argued that since it has P560,717.44 as its expenses
for the year 1953, a surplus of P347,507.01 is not
unreasonably accumulated.
3. Tried to show that investments were made with
Basilan Coconut Producers Cooperative Association
and Basilan Hospital.
4. Questioned why the examiner covered the period
from 1948-1953 when the taxable year on review
was 1953.
5. Contended that the large amounts withdrawn by or
advanced to the stockholders were advances made
in furtherance of the business purposes.
SCs response:
1. In 1948, the money was supposed to go to future
projects, but in 1953 the money was just reverted to
the general fund. This reversion made the
Government consider it for tax purposes. While an

52

TAXATION 1 | B2015
CASE DIGESTS

2.

3.

4.

5.

explanation was made saying that the fund was


spent in facilities and industrialization of the
companys industry, the SC said that this was not
sufficient explanation, quoting jurisprudence: "In
order to determine whether profits were accumulated
for the reasonable needs of the business or to avoid
the surtax upon shareholders, the controlling
intention of the taxpayer is that which is manifested
at the time of the accumulation, not subsequently
declared intentions which are merely the products of
after-thought."
There is no need to have such a large amount at the
beginning of the following year because during the
year, current assets are converted into cash and with
the income realized from the business as the year
goes, these expenses may well be taken care of.
Unreasonable
accumulation
was
even
more
pronounced because as of Dec. 31, 1953 already
P59,794.72 was spent yet as of that date there
was still a surplus of P347,507.01.
No error in the applied process because previous
accumulations should be considered in the
determination of unreasonable accumulations for the
year concerned.
As correctly held by the CTA, while certain expenses
of the corporation were credited against these
amounts, the unspent balance was retained by the
stockholders without refunding them to petitioner at
the end of each year. These advances were in fact
indirect loans to the stockholders indicating the
unreasonable accumulation of surplus beyond the
needs of the business.

SC: The unreasonable accumulation was in 1953; the


exemption mentioned above was amended more than 3
years after the period covered by the assessment.
DISPOSITIVE: Judgment modified to the extent that
petitioner is allowed its deductions for travelling and
miscellaneous expenses, but affirmed insofar as the
petitioner is liable for P2,100.67 as deficiency income tax for
1953 and P86,876.75 as 25% surtax on the unreasonably
accumulated profit of P347,507.01.

5. Petitioner: Wanted to avail of the exempting proviso in


Sec. 25 of the IRC as amended by RA 1823, approved
June 22, 1957 wherein accumulated profits or surplus if
invested in any dollar-producing or dollar-earning
industry or in the purchase of bonds issued by the
Central Bank, may not be subject to the 25% surtax.

53

Vous aimerez peut-être aussi